HESI Exit Exam Questions, Qbank Test 8

Study with our free HESI Exit Exam Questions, Qbank Test 8. The questions include answers and detailed explanations. HESI Exit nursing topics focus on critical thinking, priorities of care, health promotion and maintenance, safe and effective care, and basic care and comfort, treatments and nursing management. Achieve your best grade on the HESI Exit Exam!
Next: HESI Exit Exam Questions, Qbank Test 1
HESI Exit Exam Questions, Qbank Test 2 
HESI Exit Exam Questions, Qbank Test 3
HESI Exit Exam Questions, Qbank Test 4
HESI Exit Exam Questions, Qbank Test 5
HESI Exit Exam Questions, Qbank Test 6
HESI Exit Exam Questions, Qbank Test 7
HESI Exit Exam Questions, Qbank Test 9
HESI Exit Exam Questions, Qbank Test 10

RHEUMATORID ARTHRITIS, NCLEX, ANCC, AANP, HESI EXIT, HESI A2
Joint damage in rheumatoid arthritis

HESI Exit Question: A 55-year-old patient with a history of knee osteoarthritis is seeking a non-surgical treatment option to alleviate pain and improve joint stability. What type of orthopedic device is often prescribed in such cases?

  • A) Air splint
  • B) Elastic bandage
  • C) Rigid cast splint
  • D) Knee brace

Explanation: Knee braces can be prescribed for patients with knee osteoarthritis to provide support, reduce pain, and improve joint stability without surgery.

POP QUIZ

@qbankproacademy

4000+ Free NCLEX QUESTIONS. Go to QBankProAcademy.com FREE Qbank questions for NCLEX RN, PN, HESI Exit, Med Surg, AANP, ANCC, and HesiA2. Practice questions, quizzes, and listen to the Free Podcast. In this video, we review a question on a GUILLAIN-BARRE in Nursing Care. At QbankproAcademy.com our mission is to provide free QBanks, videos, and the most up to date test prep information for nurses. If you find our website helpful, please tell other aspiring nurses, nursing students, and professors. Please link to our site from your blogs, videos, and college websites, or share us on your favorite social media sites. Thank you for your support! Free nursing NCLEX 60-DAY CHALLENGE https://qbankpro.thinkific.com/courses/qbankpro-academy nclex Nurses Nursing aanp qbank ancc hesi medsurg qbank

♬ original sound – QBank Pro Academy – QBank Pro Academy

HESI Exit Question: A 45-year-old female patient presents with a breast lump that was detected during a routine mammogram. The physician recommends a biopsy to determine the nature of the lump. What type of biopsy is commonly performed for breast lumps?

  • A) Excisional biopsy
  • B) Incisional biopsy
  • C) Needle biopsy
  • D) Shave biopsy


Explanation: Needle biopsy is commonly performed to evaluate breast lumps. It involves using a thin needle to obtain tissue samples for analysis, often guided by imaging techniques like ultrasound.

HESI Exit Question: A 60-year-old male patient has a history of smoking and presents with chronic cough and hemoptysis (coughing up blood). A suspicious lung lesion is identified on a chest X-ray. What type of biopsy is typically recommended for lung lesions?

  • A) Excisional biopsy
  • B) Bronchoscopic biopsy
  • C) Needle biopsy
  • D) Shave biopsy

Explanation: Bronchoscopic biopsy, which involves using a flexible tube (bronchoscope) to obtain tissue samples from the lungs, is commonly used for evaluating lung lesions, especially in cases of hemoptysis and suspected malignancy.

HESI Exit Question: A 55-year-old patient presents with a skin lesion that has changed in appearance and increased in size. The healthcare provider suspects skin cancer. What type of biopsy is often performed for suspicious skin lesions?

  • A) Excisional biopsy
  • B) Incisional biopsy
  • C) Needle biopsy
  • D) Shave biopsy

The correct answer isA) Excisional biopsyExplanation: An excisional biopsy is commonly performed for suspicious skin lesions to remove the entire lesion, providing a complete specimen for examination, particularly in cases of suspected skin cancer.

HESI Exit Question: A 40-year-old patient presents with persistent abdominal pain, weight loss, and changes in bowel habits. The physician suspects gastrointestinal malignancy. What type of biopsy is frequently recommended for assessing lesions in the gastrointestinal tract?

  • A) Excisional biopsy
  • B) Endoscopic biopsy
  • C) Needle biopsy
  • D) Shave biopsy

Explanation: Endoscopic biopsy, performed during procedures like colonoscopy or upper endoscopy, is frequently used to obtain tissue samples from the gastrointestinal tract for evaluation in cases of suspected malignancy or other abnormalities.

Metronidazole may be used to treat this colon lesion, hesi exit, nclex, aanp, ancc practice questions
Colon tumor

HESI Exit Question: A 50-year-old patient presents with a thyroid nodule detected on an ultrasound. What type of biopsy is typically performed to assess thyroid nodules?

  • A) Excisional biopsy
  • B) Incisional biopsy
  • C) Fine-needle aspiration (FNA) biopsy
  • D) Shave biopsy


Explanation: Fine-needle aspiration (FNA) biopsy is commonly used to assess thyroid nodules. It involves using a thin needle to obtain a small tissue sample for analysis.

HESI Exit Question: A 35-year-old patient with a history of liver disease undergoes imaging that reveals a suspicious liver lesion. What type of biopsy is often recommended to evaluate liver lesions?

  • A) Excisional biopsy
  • B) Incisional biopsy
  • C) Needle biopsy
  • D) Shave biopsy


Explanation: Needle biopsy, such as a percutaneous liver biopsy, is frequently recommended to assess liver lesions. It involves using a thin needle to obtain liver tissue for examination.

HESI Exit Question: A 65-year-old patient presents with enlarged lymph nodes in the neck. The healthcare provider suspects lymphoma. What type of biopsy is typically recommended for diagnosing lymphoma?

  • A) Excisional biopsy
  • B) Incisional biopsy
  • C) Needle biopsy
  • D) Shave biopsy


Explanation: An excisional biopsy is often recommended for diagnosing lymphoma, as it provides a complete lymph node specimen for evaluation, aiding in accurate diagnosis and classification of the disease.

HESI Exit Question: A 30-year-old patient with a suspected bone tumor requires a tissue sample for diagnosis. What type of biopsy is commonly performed for bone tumors?

  • A) Excisional biopsy
  • B) Incisional biopsy
  • C) Needle biopsy
  • D) Shave biopsy


Explanation: An incisional biopsy is often performed for bone tumors to obtain a representative tissue sample for diagnosis while preserving as much healthy bone as possible.

HESI Exit Question: A 25-year-old patient with persistent epigastric pain is being evaluated for gastric ulcers. What type of biopsy is typically performed to assess gastric ulcers and detect Helicobacter pylori infection?

  • A) Excisional biopsy
  • B) Incisional biopsy
  • C) Needle biopsy
  • D) Endoscopic biopsy


Explanation: Endoscopic biopsy, obtained during an upper endoscopy, is commonly used to assess gastric ulcers and detect Helicobacter pylori infection in the stomach lining.

HESI Exit Question: A 20-year-old patient presents with a breast lump and is scheduled for a biopsy. What type of biopsy is often recommended for breast lesions to assess whether they are benign or malignant?

  • A) Excisional biopsy
  • B) Incisional biopsy
  • C) Needle biopsy
  • D) Shave biopsy


Explanation: Needle biopsy is commonly recommended for breast lesions to assess whether they are benign or malignant. It provides tissue samples for accurate diagnosis without the need for surgical excision.

HESI Exit Question: A 65-year-old male patient presents with left lower quadrant abdominal pain, fever, and constipation. On physical examination, he has tenderness in the lower abdomen. His vital signs are stable. What is the most likely diagnosis?

  • A) Appendicitis
  • B) Gastroenteritis
  • C) Diverticulitis
  • D) Colon cancer


Explanation: Diverticulitis is characterized by left lower quadrant abdominal pain, fever, and constipation. Physical examination may reveal tenderness in the lower abdomen.

HESI Exit Question: A 55-year-old female patient with a history of diverticulosis experiences sudden, severe lower abdominal pain, and rectal bleeding. Her blood pressure drops, and she appears pale. What is the most likely complication of diverticulosis in this patient?

  • A) Diverticulitis
  • B) Diverticular bleeding
  • C) Colonic perforation
  • D) Irritable bowel syndrome


Explanation: Sudden, severe lower abdominal pain and rectal bleeding with signs of shock are suggestive of diverticular bleeding, a serious complication of diverticulosis.

HESI Exit Question: A 70-year-old patient is diagnosed with uncomplicated diverticulitis. What is the initial management strategy for this patient?

  • A) Immediate surgery
  • B) Intravenous antibiotics
  • C) High-fiber diet
  • D) Pain management only


Correct Answer: B) Intravenous antibiotics

Explanation: Initial management of uncomplicated diverticulitis typically involves intravenous antibiotics to control infection.

colon anatomy, HESI exit practice questions
Colon anatomy, diverticular disease

HESI Exit Question: A patient with a history of recurrent diverticulitis experiences persistent lower abdominal pain, fever, and signs of peritonitis. What complication is most likely present in this patient?

  • A) Diverticular bleeding
  • B) Diverticular stricture
  • C) Diverticular abscess
  • D) Diverticular perforation


Explanation: Persistent abdominal pain, fever, and signs of peritonitis suggest diverticular perforation, a serious complication requiring urgent surgical intervention.

HESI Exit Question: A 45-year-old patient with a history of diverticulitis asks about dietary recommendations to prevent future episodes. What dietary advice should the nurse provide?

  • A) Low-fiber diet
  • B) High-fiber diet
  • C) Strict liquid diet
  • D) Avoid all dairy products


Explanation: A high-fiber diet, rich in fruits, vegetables, and whole grains, can help prevent future episodes of diverticulitis by promoting regular bowel movements and reducing pressure in the colon.

HESI Exit Question: A 30-year-old athlete presents with a painful and swollen ankle after twisting it during a soccer game. On examination, you suspect a sprained ankle. What type of splint is most appropriate for initial management?

  • A) Air splint
  • B) Ankle brace
  • C) Rigid cast splint
  • D) Elastic bandage


Explanation: An air splint provides support while allowing for swelling reduction and is often used in the initial management of sprained ankles.

HESI Exit Question: A 45-year-old construction worker presents with a suspected wrist fracture. What type of splint should be applied to stabilize the wrist while awaiting further evaluation?

  • A) Rigid cast splint
  • B) Elastic bandage
  • C) Wrist brace
  • D) Air splint


Explanation: A rigid cast splint is used to immobilize fractures and is suitable for stabilizing a suspected wrist fracture.

NCLEX Musculoskeletal Questions
Fractured Leg With External Fixator

HESI Exit Question: A 60-year-old patient presents with a strained lower back due to heavy lifting. What type of splint or support device can help alleviate pain and provide support for this condition?

  • A) Rigid cast splint
  • B) Lumbar corset
  • C) Elastic bandage
  • D) Air splint


Explanation: A lumbar corset provides support for strained lower back muscles and can help alleviate pain in such cases.

HESI Exit Question: A 25-year-old patient presents with a suspected forearm fracture. What type of splint should be applied to immobilize the forearm?

  • A) Elastic bandage
  • B) Rigid cast splint
  • C) Forearm sleeve
  • D) Air splint

Explanation: A forearm sleeve or forearm splint is used to immobilize forearm fractures and provide support.

HESI Exit Question: A 40-year-old patient with a suspected ankle fracture needs immediate immobilization. Which type of splint or support device should be applied in the field before transferring the patient to the hospital?

  • A) Air splint
  • B) Elastic bandage
  • C) Ankle brace
  • D) Rigid cast splint


Correct Answer: A) Air splint

Explanation: An air splint is portable and can be applied in the field to provide immediate immobilization for suspected fractures.

hesi exit, nclex, aanp, ancc practice questions
Inpatient rehabilitation nursing

HESI Exit Question: A 55-year-old patient presents with a suspected shoulder dislocation. What type of splint or support device should be used to temporarily immobilize the shoulder?

  • A) Shoulder sling
  • B) Rigid cast splint
  • C) Elastic bandage
  • D) Air splint


Correct Answer: A) Shoulder sling

Explanation: A shoulder sling can provide temporary immobilization and support for a suspected shoulder dislocation.

HESI Exit Question: A 35-year-old patient presents with a fractured femur. What type of splint or support device should be applied to immobilize the fractured leg?

  • A) Air splint
  • B) Elastic bandage
  • C) Rigid cast splint
  • D) Knee brace

Correct Answer: C) Rigid cast splint

Explanation: A rigid cast splint is typically used to immobilize a fractured femur and provide stability.

HESI Exit Question: A 50-year-old patient presents with a suspected wrist sprain. What type of splint should be applied to provide support and prevent further injury?

  • A) Elastic bandage
  • B) Rigid cast splint
  • C) Wrist brace
  • D) Air splint


Correct Answer: C) Wrist brace

Explanation: A wrist brace can provide support and prevent further injury in cases of wrist sprains.

HESI Exit Question: A 70-year-old patient presents with a suspected rib fracture due to a fall. What type of splint or support device should be used to stabilize the ribcage and reduce pain?

  • A) Rigid cast splint
  • B) Elastic bandage
  • C) Rib belt
  • D) Air splint


Correct Answer: C) Rib belt

Explanation: A rib belt can provide support and stabilization for rib fractures and help reduce pain during breathing.

hesi exit, nclex, aanp, ancc practice questions
A nurses healing hands

HESI Exit Question: A 20-year-old patient presents with a suspected ankle strain after a fall. What type of splint or support device should be used to provide support and assist with ambulation?

  • A) Rigid cast splint
  • B) Ankle brace
  • C) Elastic bandage
  • D) Air splint


Correct Answer: B) Ankle brace

Explanation: An ankle brace can provide support for ankle strains and assist with ambulation while protecting the injured area.

HESI Exit Question: A 45-year-old patient with a fractured forearm requires temporary immobilization. Which type of splint is commonly used to provide support and stability for this injury?

  • A) Air splint
  • B) Elastic bandage
  • C) Rigid cast splint
  • D) Knee brace


Explanation: An air splint is often used for temporary immobilization of a fractured forearm. It provides support while allowing for adjustments and swelling reduction.

hesi exit, nclex, aanp, ancc practice questions
Success!

HESI Exit Question: A 60-year-old patient is recovering from a leg fracture and has just had their cast removed. What type of device can assist in maintaining joint mobility and preventing muscle atrophy during the rehabilitation process?

  • A) Air splint
  • B) Rigid cast splint
  • C) Knee brace
  • D) Ankle brace


Explanation: A knee brace can help maintain joint mobility and muscle strength during rehabilitation after a leg fracture and cast removal.

HESI Exit Question: A 30-year-old athlete has been diagnosed with a mild ankle sprain. What type of device is commonly recommended for managing ankle sprains and providing support?

  • A) Rigid cast splint
  • B) Elastic bandage
  • C) Air splint
  • D) Ankle brace


Explanation: Ankle braces are commonly recommended for managing mild ankle sprains as they provide support and help prevent further injury.

HESI Exit Question: A 25-year-old patient with a fractured wrist has recently had a cast applied. What should be emphasized to the patient regarding care and maintenance of the cast?

  • A) Keep the cast exposed to air for better healing
  • B) Apply lotion under the cast to prevent dryness
  • C) Avoid inserting objects inside the cast
  • D) Remove the cast for daily cleaning


Explanation: Patients should be advised to avoid inserting objects inside the cast, as this can lead to complications and skin problems. Keeping the cast dry is also important.

hesi exit, nclex, aanp, ancc practice questions, pulse oximeter
Inpatient with pulse oximeter

HESI Exit Question: A 35-year-old patient with a history of lower back pain is recommended for a lumbar brace to provide support and alleviate discomfort. What is the primary purpose of a lumbar brace in such cases?

  • A) To restrict all movement of the lower back
  • B) To provide warmth to the lower back
  • C) To maintain proper posture
  • D) To promote flexibility


Explanation: A lumbar brace is primarily used to help maintain proper posture and provide support to the lower back, reducing discomfort associated with lower back pain.

HESI Exit Question: A 50-year-old patient with a fractured femur has had a rigid cast applied for immobilization. What important nursing intervention should be implemented to prevent complications?

  • A) Encourage frequent cast removal for inspection
  • B) Elevate the affected leg above heart level
  • C) Apply pressure to the cast to alleviate itching
  • D) Monitor for signs of neurovascular compromise


Explanation: It’s essential to monitor for signs of neurovascular compromise, such as changes in circulation and sensation, to prevent complications in patients with a rigid cast on their femur.

HESI Exit Question: A 70-year-old patient with a history of wrist arthritis is prescribed a wrist splint. What is the primary purpose of a wrist splint in managing wrist arthritis?

  • A) To restrict all movement of the wrist
  • B) To provide warmth to the wrist
  • C) To maintain proper wrist alignment
  • D) To promote wrist flexibility


Explanation: A wrist splint is primarily used to maintain proper wrist alignment and reduce discomfort in patients with wrist arthritis.

HESI Exit Question: A 20-year-old patient with a knee injury is recommended a hinged knee brace for rehabilitation. What is the advantage of a hinged knee brace in this scenario?

  • A) It completely immobilizes the knee
  • B) It provides warmth to the knee
  • C) It allows controlled range of motion
  • D) It promotes flexibility


Explanation: A hinged knee brace allows controlled range of motion in the knee joint, which is important for rehabilitation and preventing stiffness.

HESI Exit Question: A 65-year-old patient with a foot injury is prescribed a walking boot. What is the primary purpose of a walking boot in managing foot injuries?

  • A) To completely immobilize the foot
  • B) To provide warmth to the foot
  • C) To maintain proper foot alignment
  • D) To promote foot flexibility


Explanation: A walking boot is primarily used to maintain proper foot alignment and support during the healing process of foot injuries.

HESI Exit Question: A 45-year-old female patient presents with a breast lump that was detected during a routine mammogram. The physician recommends a biopsy to determine the nature of the lump. What type of biopsy is commonly performed for breast lumps?

  • A) Excisional biopsy
  • B) Incisional biopsy
  • C) Needle biopsy
  • D) Shave biopsy


Explanation: Needle biopsy is commonly performed to evaluate breast lumps. It involves using a thin needle to obtain tissue samples for analysis, often guided by imaging techniques like ultrasound.

hesi exit, nclex, aanp, ancc practice questions,Respiratory NCLEX Questions, PNEUMONIA
Chest X-ray showing a right lung mass

HESI Exit Question: A 60-year-old male patient has a history of smoking and presents with chronic cough and hemoptysis (coughing up blood). A suspicious lung lesion is identified on a chest X-ray. What type of biopsy is typically recommended for lung lesions?

  • A) Excisional biopsy
  • B) Bronchoscopic biopsy
  • C) Needle biopsy
  • D) Shave biopsy


Explanation: Bronchoscopic biopsy, which involves using a flexible tube (bronchoscope) to obtain tissue samples from the lungs, is commonly used for evaluating lung lesions, especially in cases of hemoptysis and suspected malignancy.

HESI Exit Question: A 55-year-old patient presents with a skin lesion that has changed in appearance and increased in size. The healthcare provider suspects skin cancer. What type of biopsy is often performed for suspicious skin lesions?

  • A) Excisional biopsy
  • B) Incisional biopsy
  • C) Needle biopsy
  • D) Shave biopsy


Explanation: An excisional biopsy is commonly performed for suspicious skin lesions to remove the entire lesion, providing a complete specimen for examination, particularly in cases of suspected skin cancer.

HESI Exit Question: A 40-year-old patient presents with persistent abdominal pain, weight loss, and changes in bowel habits. The physician suspects gastrointestinal malignancy. What type of biopsy is frequently recommended for assessing lesions in the gastrointestinal tract?

  • A) Excisional biopsy
  • B) Endoscopic biopsy
  • C) Needle biopsy
  • D) Shave biopsy


Explanation: Endoscopic biopsy, performed during procedures like colonoscopy or upper endoscopy, is frequently used to obtain tissue samples from the gastrointestinal tract for evaluation in cases of suspected malignancy or other abnormalities.

HESI Exit Question: A 50-year-old patient presents with a thyroid nodule detected on an ultrasound. What type of biopsy is typically performed to assess thyroid nodules?

  • A) Excisional biopsy
  • B) Incisional biopsy
  • C) Fine-needle aspiration (FNA) biopsy
  • D) Shave biopsy


Explanation: Fine-needle aspiration (FNA) biopsy is commonly used to assess thyroid nodules. It involves using a thin needle to obtain a small tissue sample for analysis.

HESI Exit Question: A 35-year-old patient with a history of liver disease undergoes imaging that reveals a suspicious liver lesion. What type of biopsy is often recommended to evaluate liver lesions?

  • A) Excisional biopsy
  • B) Incisional biopsy
  • C) Needle biopsy
  • D) Shave biopsy


Explanation: Needle biopsy, such as a percutaneous liver biopsy, is frequently recommended to assess liver lesions. It involves using a thin needle to obtain liver tissue for examination.

HESI Exit Question: A 65-year-old patient presents with enlarged lymph nodes in the neck. The healthcare provider suspects lymphoma. What type of biopsy is typically recommended for diagnosing lymphoma?

  • A) Excisional biopsy
  • B) Incisional biopsy
  • C) Needle biopsy
  • D) Shave biopsy


Explanation: An excisional biopsy is often recommended for diagnosing lymphoma, as it provides a complete lymph node specimen for evaluation, aiding in accurate diagnosis and classification of the disease.

HESI Exit Question: A 30-year-old patient with a suspected bone tumor requires a tissue sample for diagnosis. What type of biopsy is commonly performed for bone tumors?

  • A) Excisional biopsy
  • B) Incisional biopsy
  • C) Needle biopsy
  • D) Shave biopsy

Explanation: An incisional biopsy is often performed for bone tumors to obtain a representative tissue sample for diagnosis while preserving as much healthy bone as possible.

HESI Exit Question: A 25-year-old patient with persistent epigastric pain is being evaluated for gastric ulcers. What type of biopsy is typically performed to assess gastric ulcers and detect Helicobacter pylori infection?

  • A) Excisional biopsy
  • B) Incisional biopsy
  • C) Needle biopsy
  • D) Endoscopic biopsy


Explanation: Endoscopic biopsy, obtained during an upper endoscopy, is commonly used to assess gastric ulcers and detect Helicobacter pylori infection in the stomach lining.

NCLEX Stomach Questions, hesi exit, nclex, aanp, ancc practice questions
Stomach anatomy affected by GERD and peptic ulcer disease

HESI Exit Question: A 20-year-old patient presents with a breast lump and is scheduled for a biopsy. What type of biopsy is often recommended for breast lesions to assess whether they are benign or malignant?

  • A) Excisional biopsy
  • B) Incisional biopsy
  • C) Needle biopsy
  • D) Shave biopsy


Explanation: Needle biopsy is commonly recommended for breast lesions to assess whether they are benign or malignant. It provides tissue samples for accurate diagnosis without the need for surgical excision.

HESI Exit Question: A 75-year-old diabetic patient presents with a non-healing ulcer on the lower leg. The wound is red, warm, and has a foul odor. What type of skin ulcer is most likely present?

  • A) Arterial ulcer
  • B) Venous ulcer
  • C) Diabetic ulcer
  • D) Pressure ulcer


Explanation: The characteristics of a non-healing ulcer in a diabetic patient, such as warmth, foul odor, and slow healing, are indicative of a diabetic ulcer. Diabetic ulcers are often associated with neuropathy and vascular complications.

HESI Exit Question: A 60-year-old patient presents with a painful, shallow ulcer on the medial aspect of the ankle. The surrounding skin is erythematous, and the wound appears to be weeping clear fluid. What type of skin ulcer is most likely present?

  • A) Arterial ulcer
  • B) Venous ulcer
  • C) Diabetic ulcer
  • D) Pressure ulcer


Explanation: The characteristics of a painful, shallow ulcer with erythema and clear fluid weeping suggest a venous ulcer. Venous ulcers often occur on the lower legs and are associated with venous insufficiency.

HESI Exit Question: A 45-year-old patient with a history of smoking presents with a painful ulcer on the toes. The ulcer appears black with well-defined borders and minimal exudate. What type of skin ulcer is most likely present?

  • A) Arterial ulcer
  • B) Venous ulcer
  • C) Diabetic ulcer
  • D) Pressure ulcer


Explanation: The black appearance, well-defined borders, and minimal exudate are characteristic of an arterial ulcer. Arterial ulcers often occur due to inadequate blood supply, as seen in peripheral arterial disease (PAD).

HESI Exit Question: A bedridden 70-year-old patient develops a painful ulcer over the sacral area. The ulcer has a shallow appearance with localized tissue damage and minimal drainage. What type of skin ulcer is most likely present?

  • A) Arterial ulcer
  • B) Venous ulcer
  • C) Diabetic ulcer
  • D) Pressure ulcer


Explanation: The development of a pressure ulcer in a bedridden patient is common, especially over bony prominences like the sacral area. Pressure ulcers result from prolonged pressure on the skin, leading to tissue damage.

HESI Exit Question: A 55-year-old patient with a history of autoimmune disease presents with a deep, irregularly shaped ulcer on the lower leg. The ulcer is associated with undermined edges and granulation tissue. What type of skin ulcer is most likely present?

  • A) Arterial ulcer
  • B) Venous ulcer
  • C) Diabetic ulcer
  • D) Pyoderma gangrenosum


Explanation: The deep, irregular shape, undermined edges, and granulation tissue suggest pyoderma gangrenosum, which is associated with autoimmune conditions and typically requires specialized care.

HESI Exit Question: A 35-year-old patient with a history of obesity and immobility develops a painful, deep ulcer on the medial aspect of the calf. The ulcer has irregular edges and purulent discharge. What type of skin ulcer is most likely present?

  • A) Arterial ulcer
  • B) Venous ulcer
  • C) Diabetic ulcer
  • D) Pressure ulcer


Explanation: The characteristics of a painful ulcer with irregular edges and purulent discharge are suggestive of a venous ulcer, which can occur in individuals with obesity and immobility due to venous insufficiency.

HESI Exit Question: A 50-year-old patient with a history of trauma presents with a deep ulcer at the site of a previous injury. The ulcer has necrotic tissue and foul odor. What type of skin ulcer is most likely present?

  • A) Arterial ulcer
  • B) Venous ulcer
  • C) Diabetic ulcer
  • D) Pressure ulcer

Explanation: The presence of necrotic tissue, foul odor, and a history of trauma suggests a pressure ulcer. Pressure ulcers can develop at injury sites, especially if there is ongoing pressure and inadequate wound care.

HESI Exit Question: A 30-year-old patient presents with a painful ulcer on the dorsum of the foot. The ulcer has well-defined edges and moderate exudate. What type of skin ulcer is most likely present?

  • A) Arterial ulcer
  • B) Venous ulcer
  • C) Neuropathic ulcer
  • D) Pressure ulcer

Correct Answer: C) Neuropathic ulcer

Explanation: The well-defined edges, location on the foot dorsum, and moderate exudate are indicative of a neuropathic ulcer. Neuropathic ulcers commonly occur in patients with neuropathy, such as those with diabetes.

HESI Exit Question: A 25-year-old patient presents with a rapidly progressing, painful ulcer on the hand after a minor injury. The ulcer is surrounded by erythema and has purulent drainage. What type of skin ulcer is most likely present?

  • A) Arterial ulcer
  • B) Venous ulcer
  • C) Diabetic ulcer
  • D) Infectious ulcer

Correct Answer: D) Infectious ulcer

Explanation: The rapidly progressing nature, surrounding erythema, purulent drainage, and history of minor injury suggest an infectious ulcer. Timely assessment and appropriate treatment are crucial to prevent further complications.

HESI Exit Question: A 65-year-old patient with a history of congestive heart failure presents with a painful ulcer on the lower leg. The ulcer is associated with significant edema and hemosiderin staining. What type of skin ulcer is most likely present?

  • A) Arterial ulcer
  • B) Venous ulcer
  • C) Diabetic ulcer
  • D) Mixed etiology ulcer

Correct Answer: B) Venous ulcer

Explanation: The presence of significant edema, hemosiderin staining, and the location on the lower leg are characteristic of a venous ulcer, often seen in patients with congestive heart failure and venous insufficiency.

HESI Exit Question: A 45-year-old patient underwent abdominal surgery and developed a large surgical wound with moderate drainage. Which wound treatment method helps promote wound healing, reduces the risk of infection, and manages exudate effectively?

  • A) Traditional dry gauze dressing
  • B) Wet-to-dry dressing
  • C) Negative pressure wound therapy (NPWT)
  • D) Hydrocolloid dressing

Correct Answer: C) Negative pressure wound therapy (NPWT)

Explanation: Negative pressure wound therapy (NPWT) is an advanced wound treatment method that creates a controlled environment, removing excess exudate, promoting tissue granulation, and reducing the risk of infection. It is particularly effective for large surgical wounds.

HESI Exit Question: A 60-year-old patient has a pressure ulcer on the sacral area. The wound is deep and contains slough and necrotic tissue. Which wound debridement method is most appropriate for this stage of wound healing?

  • A) Surgical debridement
  • B) Autolytic debridement
  • C) Enzymatic debridement
  • D) Maggot therapy

Correct Answer: A) Surgical debridement

Explanation: In cases with deep pressure ulcers containing slough and necrotic tissue, surgical debridement by a healthcare provider is often necessary to remove non-viable tissue and promote wound healing.

HESI Exit Question: A 55-year-old diabetic patient has a non-healing foot ulcer with moderate exudate. What type of dressing is suitable for managing this diabetic foot ulcer while providing a moist wound environment?

  • A) Traditional dry gauze dressing
  • B) Wet-to-dry dressing
  • C) Hydrocolloid dressing
  • D) Alginate dressing

Correct Answer: D) Alginate dressing

Explanation: Alginate dressings are highly absorbent and suitable for diabetic foot ulcers with moderate exudate. They maintain a moist wound environment, facilitate autolytic debridement, and promote healing.

HESI Exit Question: A 40-year-old patient has a traumatic laceration with minimal exudate. What type of dressing is typically used for managing clean, closed surgical wounds or superficial injuries?

  • A) Traditional dry gauze dressing
  • B) Wet-to-dry dressing
  • C) Negative pressure wound therapy (NPWT)
  • D) Hydrocolloid dressing

Correct Answer: A) Traditional dry gauze dressing

Explanation: For clean, closed surgical wounds or superficial injuries with minimal exudate, traditional dry gauze dressing is commonly used to provide protection and facilitate healing.

HESI Exit Question: A 50-year-old patient with a leg wound presents with signs of infection, including redness, warmth, and purulent discharge. Which type of dressing may be appropriate for managing infected wounds and promoting a clean wound environment?

  • A) Traditional dry gauze dressing
  • B) Wet-to-dry dressing
  • C) Silver-impregnated dressing
  • D) Alginate dressing

Correct Answer: C) Silver-impregnated dressing

Explanation: Silver-impregnated dressings have antimicrobial properties and are often used for infected wounds to reduce bacterial load and promote a clean wound environment.

HESI Exit Question: A 35-year-old patient has a complex surgical wound with exposed bone and tendon. Which wound dressing option provides protection, supports tissue growth, and facilitates healing in such cases?

  • A) Traditional dry gauze dressing
  • B) Wet-to-dry dressing
  • C) Negative pressure wound therapy (NPWT)
  • D) Collagen dressing

Correct Answer: D) Collagen dressing

Explanation: Collagen dressings promote tissue growth, provide protection, and support healing, making them suitable for complex surgical wounds with exposed bone and tendon.

HESI Exit Question: A 65-year-old patient with a leg ulcer has significant slough and necrotic tissue in the wound bed. What type of debridement method is a non-invasive option for removing necrotic tissue in this wound?

  • A) Surgical debridement
  • B) Autolytic debridement
  • C) Enzymatic debridement
  • D) Mechanical debridement

Correct Answer: B) Autolytic debridement

Explanation: Autolytic debridement is a non-invasive process that utilizes the body’s natural enzymes and moisture to gradually break down and remove necrotic tissue, making it suitable for wounds with slough.

HESI Exit Question: A 30-year-old patient has a surgical wound with minimal exudate and a desire for minimal dressing changes. Which type of dressing provides an optimal environment for moist wound healing while requiring fewer changes?

  • A) Traditional dry gauze dressing
  • B) Wet-to-dry dressing
  • C) Hydrocolloid dressing
  • D) Alginate dressing

Correct Answer: C) Hydrocolloid dressing

Explanation: Hydrocolloid dressings maintain a moist wound environment, support autolytic debridement, and require fewer changes compared to traditional dry gauze dressings, making them suitable for wounds with minimal exudate.

HESI Exit Question: A 25-year-old patient has a complex wound with exposed hardware following orthopedic surgery. Which wound treatment option provides protection, promotes tissue granulation, and minimizes pain during dressing changes?

  • A) Traditional dry gauze dressing
  • B) Wet-to-dry dressing
  • C) Negative pressure wound therapy (NPWT)
  • D) Silicone foam dressing

Correct Answer: D) Silicone foam dressing

Explanation: Silicone foam dressings provide protection, absorb exudate, support tissue granulation, and minimize pain during dressing changes, making them suitable for wounds with exposed hardware.

HESI Exit Question: A 65-year-old patient with a large, complex abdominal wound following surgery requires wound protection and support for tissue healing. Which advanced wound treatment option offers both protection and continuous monitoring capabilities?

  • A) Traditional dry gauze dressing
  • B) Wet-to-dry dressing
  • C) Negative pressure wound therapy (NPWT)
  • D) Honey-impregnated dressing

Correct Answer: C) Negative pressure wound therapy (NPWT)

Explanation: Negative pressure wound therapy (NPWT) offers wound protection, continuous monitoring, and controlled suction to promote tissue healing, making it a valuable option for large, complex wounds.

HESI Exit Question: A 68-year-old patient presents with a history of diabetes and complains of a persistent, non-blanchable redness over the sacral area. Which type of skin assessment finding is indicative of this condition?

  • A) Ecchymosis
  • B) Purpura
  • C) Blanching erythema
  • D) Non-blanchable erythema

Correct Answer: D) Non-blanchable erythema

Explanation: The presence of non-blanchable erythema indicates a Stage 1 pressure ulcer. In patients with risk factors like diabetes, prevention and early detection are essential to prevent further skin damage.

HESI Exit Question: A 42-year-old patient presents with a rash on the forearm characterized by raised, itchy, red welts. Which type of skin assessment finding is consistent with this presentation?

  • A) Petechiae
  • B) Papules
  • C) Telangiectasia
  • D) Purpura

Correct Answer: B) Papules

Explanation: The description of raised, itchy, red welts on the forearm is characteristic of papules, which are a type of skin lesion often seen in conditions like urticaria (hives).

HESI Exit Question: A 55-year-old patient presents with dilated blood vessels visible on the cheeks and nose. Which type of skin assessment finding is typically associated with this condition?

  • A) Ecchymosis
  • B) Petechiae
  • C) Telangiectasia
  • D) Purpura

Correct Answer: C) Telangiectasia

Explanation: Telangiectasia refers to dilated blood vessels visible on the skin’s surface, commonly seen in conditions like rosacea.

HESI Exit Question: A 35-year-old patient presents with small, pinpoint-sized, purple-red spots on the skin. Which type of skin assessment finding is indicative of this condition?

  • A) Ecchymosis
  • B) Papules
  • C) Petechiae
  • D) Purpura

Correct Answer: C) Petechiae

Explanation: Petechiae are small, pinpoint-sized, purple-red spots on the skin, often associated with bleeding disorders or capillary fragility.

HESI Exit Question: A 60-year-old patient presents with a circular, fluid-filled lesion on the forearm that is painful and itchy. Which type of skin assessment finding is consistent with this presentation?

  • A) Macule
  • B) Pustule
  • C) Vesicle
  • D) Plaque

Correct Answer: C) Vesicle

Explanation: A vesicle is a circular, fluid-filled lesion on the skin, often associated with conditions like herpes simplex or contact dermatitis.

HESI Exit Question: A 48-year-old patient presents with thickened, scaly skin on the elbows and knees. Which type of skin assessment finding is commonly associated with this presentation?

  • A) Excoriation
  • B) Lichenification
  • C) Ulceration
  • D) Fissure

Correct Answer: B) Lichenification

Explanation: Lichenification is characterized by thickened, scaly skin, often seen in chronic skin conditions like eczema or psoriasis.

HESI Exit Question: A 28-year-old patient presents with a wound that has exposed muscle and bone. Which type of wound assessment finding indicates that this wound has reached the deepest tissue level?

  • A) Erythema
  • B) Maceration
  • C) Tunneling
  • D) Slough

Correct Answer: C) Tunneling

Explanation: Tunneling is a wound assessment finding that indicates the wound has reached the deepest tissue level, with channels extending into the surrounding tissue.

HESI Exit Question: A 70-year-old patient with a history of venous insufficiency presents with a leg ulcer. The wound has a yellow, stringy, and fibrinous appearance. Which type of wound assessment finding is indicative of this wound characteristic?

  • A) Eschar
  • B) Slough
  • C) Granulation tissue
  • D) Excoriation

Correct Answer: B) Slough

Explanation: Slough is a yellow, stringy, fibrinous material often seen in non-viable tissue within a wound. It should be removed to promote wound healing.

HESI Exit Question: A 25-year-old patient presents with a blister-like lesion filled with clear fluid. Which type of skin assessment finding is consistent with this presentation?

  • A) Pustule
  • B) Macule
  • C) Vesicle
  • D) Plaque

Correct Answer: C) Vesicle

Explanation: A vesicle is a blister-like lesion filled with clear fluid, often seen in conditions like herpes zoster (shingles).

HESI Exit Question: A 62-year-old patient presents with a wound that has a thick, hard, black covering. Which type of wound assessment finding is indicative of this wound characteristic?

  • A) Eschar
  • B) Slough
  • C) Granulation tissue
  • D) Excoriation

Correct Answer: A) Eschar

Explanation: Eschar is a thick, hard, black or brown covering often seen in wounds with necrotic tissue. It requires debridement for wound healing to occur.

HESI Exit Question: A 35-year-old patient reports severe pain, pallor, and coolness in the left lower leg after sustaining an injury. Which neurovascular assessment finding is most concerning in this case?

  • A) Warmth
  • B) Paresthesia
  • C) Pallor
  • D) Capillary refill of 3 seconds

Correct Answer: C) Pallor

Explanation: Pallor in the affected limb suggests poor perfusion, which is a concerning neurovascular finding. Warmth is expected in a well-perfused limb, and paresthesia may indicate nerve involvement. A capillary refill of 3 seconds is delayed, but pallor takes precedence in this scenario.

HESI Exit Question: A 42-year-old patient with a history of hypertension experiences sudden, severe headache, blurred vision, and left-sided weakness. Which neurovascular assessment finding is most indicative of a potential stroke?

  • A) Pupil dilation
  • B) Unequal chest expansion
  • C) Decreased level of consciousness
  • D) Paresthesia

Correct Answer: C) Decreased level of consciousness

Explanation: A decreased level of consciousness is a critical neurovascular finding in a patient with potential stroke. Pupil dilation, unequal chest expansion, and paresthesia are important but secondary considerations in this context.

HESI Exit Question: A 50-year-old patient with diabetes complains of numbness and tingling in the feet. Which neurovascular assessment finding is consistent with neuropathy associated with diabetes?

  • A) Increased pedal pulses
  • B) Warm and dry feet
  • C) Loss of protective sensation
  • D) Absent ankle reflexes

Correct Answer: C) Loss of protective sensation

Explanation: Loss of protective sensation in the feet is a common finding in diabetic neuropathy. Increased pedal pulses, warm and dry feet, and absent ankle reflexes may be observed in other neurovascular conditions.

HESI Exit Question: A 28-year-old patient presents with a leg injury following a motor vehicle accident. Which neurovascular assessment finding is most indicative of a potential compartment syndrome?

  • A) Strong pedal pulses
  • B) Erythema
  • C) Swelling
  • D) Paresthesia

Correct Answer: D) Paresthesia

Explanation: Paresthesia (abnormal sensation) in the affected limb is a concerning neurovascular finding in potential compartment syndrome. Strong pedal pulses, erythema, and swelling may be present but do not indicate compartment syndrome as clearly as paresthesia.

HESI Exit Question: A 60-year-old patient with a history of deep vein thrombosis (DVT) reports calf pain and swelling. Which neurovascular assessment finding may indicate a potential DVT?

  • A) Warmth and erythema
  • B) Bilateral ankle edema
  • C) Absent pedal pulses
  • D) Paresthesia

Correct Answer: A) Warmth and erythema

Explanation: Warmth and erythema in the calf may indicate inflammation associated with a DVT. Bilateral ankle edema, absent pedal pulses, and paresthesia are less specific for DVT.

HESI Exit Question: A 45-year-old patient with a history of smoking presents with intermittent claudication and leg pain when walking. Which neurovascular assessment finding is consistent with peripheral arterial disease (PAD)?

  • A) Cool and pale extremities
  • B) Bilateral ankle edema
  • C) Absent pedal pulses
  • D) Paresthesia

Correct Answer: A) Cool and pale extremities

Explanation: Cool and pale extremities are indicative of poor peripheral perfusion, as seen in peripheral arterial disease (PAD). Bilateral ankle edema, absent pedal pulses, and paresthesia may be observed in other conditions.

HESI Exit Question: A 55-year-old patient presents with muscle weakness, fatigue, and difficulty swallowing. Which neurovascular assessment finding may suggest myasthenia gravis?

  • A) Loss of protective sensation
  • B) Decreased level of consciousness
  • C) Absent deep tendon reflexes
  • D) Muscle weakness that worsens with activity

Correct Answer: D) Muscle weakness that worsens with activity

Explanation: Muscle weakness that worsens with activity is a characteristic finding in myasthenia gravis, an autoimmune neuromuscular disorder. Loss of protective sensation, decreased level of consciousness, and absent deep tendon reflexes are not typical features of myasthenia gravis.

HESI Exit Question: A 68-year-old patient with a history of stroke has difficulty speaking and swallowing. Which neurovascular assessment finding is most relevant to this patient’s condition?

  • A) Warm and dry skin
  • B) Absent ankle reflexes
  • C) Decreased level of consciousness
  • D) Facial asymmetry

Correct Answer: D) Facial asymmetry

Explanation: Facial asymmetry is a classic sign of stroke or cerebrovascular accident (CVA). Warm and dry skin, absent ankle reflexes, and decreased level of consciousness are less specific to this condition.

HESI Exit Question: A 62-year-old patient with a history of osteoarthritis experiences joint pain, stiffness, and limited range of motion in the fingers. Which neurovascular assessment finding is most relevant to this patient’s condition?

  • A) Absent pedal pulses
  • B) Decreased level of consciousness
  • C) Swelling and erythema
  • D) Heberden’s nodes

Correct Answer: D) Heberden’s nodes

Explanation: Heberden’s nodes are bony enlargements seen in the distal finger joints and are associated with osteoarthritis. Absent pedal pulses, decreased level of consciousness, and swelling and erythema are not characteristic of this condition.

HESI Exit Question: A 72-year-old patient presents with severe leg pain, swelling, and redness. Which neurovascular assessment finding is most concerning in this patient and may suggest deep vein thrombosis (DVT)?

  • A) Paresthesia
  • B) Decreased level of consciousness
  • C) Absent pedal pulses
  • D) Homans’ sign

Correct Answer: D) Homans’ sign

Explanation: Homans’ sign, which is pain in the calf upon dorsiflexion of the foot, is a potential indicator of deep vein thrombosis (DVT). Paresthesia, decreased level of consciousness, and absent pedal pulses are not specific for DVT.

HESI Exit Question: A 70-year-old patient is admitted with a fractured hip following a fall. The nurse observes the patient’s right leg appearing shorter and externally rotated. What mobility assessment finding is indicative of a possible hip fracture?

  • A) Decreased muscle strength
  • B) Limited range of motion
  • C) Shortened leg with external rotation
  • D) Gait disturbance

Correct Answer: C) Shortened leg with external rotation

Explanation: A shortened leg with external rotation is a classic sign of a hip fracture. This finding occurs due to the displacement of the fractured bone. It’s crucial for the nurse to recognize this sign promptly to initiate appropriate care, including immobilization and pain management.

HESI Exit Question: A 45-year-old patient with a history of diabetes reports numbness and tingling in the feet. The nurse observes bilateral foot ulcers and loss of sensation. Which mobility assessment finding is associated with diabetic neuropathy?

  • A) Decreased muscle strength
  • B) Limited range of motion
  • C) Loss of protective sensation
  • D) Gait disturbance

Correct Answer: C) Loss of protective sensation

Explanation: Loss of protective sensation in the feet is a characteristic finding in diabetic neuropathy. Patients with this condition are at increased risk of foot ulcers and injury due to impaired sensation, underscoring the importance of regular foot assessments and education on foot care.

HESI Exit Question: A 58-year-old patient who underwent knee replacement surgery reports difficulty in bending the knee and experiencing pain during movement. Which mobility assessment finding is relevant to the evaluation of knee joint function post-surgery?

  • A) Decreased muscle strength
  • B) Limited range of motion
  • C) Loss of protective sensation
  • D) Gait disturbance

Correct Answer: B) Limited range of motion

Explanation: Limited range of motion in the knee is a significant mobility assessment finding following knee replacement surgery. It can indicate surgical complications, such as joint stiffness or adhesions, which need to be addressed through rehabilitation and physical therapy.

HESI Exit Question: A 65-year-old patient with a history of stroke presents with muscle weakness and an unsteady gait. The nurse observes the patient’s difficulty in coordinating leg movements during walking. Which mobility assessment finding may be associated with this presentation?

  • A) Decreased muscle strength
  • B) Limited range of motion
  • C) Loss of protective sensation
  • D) Gait disturbance

Correct Answer: D) Gait disturbance

Explanation: A gait disturbance is a common mobility assessment finding in patients with stroke-related motor deficits. Impaired coordination and muscle weakness can lead to an unsteady gait, making fall risk assessment and mobility interventions crucial in stroke care.

HESI Exit Question: A 55-year-old patient with obesity reports chronic lower back pain and difficulty in standing upright for extended periods. The nurse observes an exaggerated lumbar curve in the patient’s posture. What mobility assessment finding is suggestive of this presentation?

  • A) Decreased muscle strength
  • B) Limited range of motion
  • C) Loss of protective sensation
  • D) Postural abnormalities

Correct Answer: D) Postural abnormalities

Explanation: Exaggerated lumbar curve or postural abnormalities can contribute to lower back pain and difficulty in standing upright. These findings are associated with poor posture and musculoskeletal issues, emphasizing the importance of posture assessment and education in pain management.

HESI Exit Question: A 75-year-old patient has a history of falls and reports dizziness when standing up. Which intervention is most important for fall prevention in this patient?

  • A) Encourage the use of a cane
  • B) Initiate physical therapy for balance training
  • C) Educate on proper footwear
  • D) Conduct a home safety assessment

Correct Answer: D) Conduct a home safety assessment

Explanation: Conducting a home safety assessment is crucial in identifying potential fall hazards and making necessary modifications to ensure a safe environment. While the other interventions are important, addressing the patient’s home environment takes precedence in fall prevention.

HESI Exit Question: A 68-year-old patient is taking multiple medications and has a history of falls. Which nursing action is essential in preventing falls related to medication use?

  • A) Administer medications at convenient times
  • B) Encourage the patient to self-administer medications
  • C) Conduct medication reconciliation
  • D) Provide a daily medication organizer

Correct Answer: C) Conduct medication reconciliation

Explanation: Conducting medication reconciliation helps identify potential drug interactions, side effects, and the need for medication adjustments. It is a vital step in preventing falls related to medication use by ensuring the patient is on appropriate medications with the lowest fall risk.

HESI Exit Question: An 80-year-old patient with mobility limitations is at risk for falls. Which nursing intervention promotes fall prevention in this patient?

  • A) Encourage independent mobility
  • B) Administer sedative medications at bedtime
  • C) Use bed alarms for constant monitoring
  • D) Implement a structured exercise program

Correct Answer: D) Implement a structured exercise program

Explanation: Implementing a structured exercise program is essential in improving strength, balance, and mobility in older adults. This intervention can reduce fall risk and promote overall well-being. Encouraging independent mobility is beneficial but should be done safely with appropriate exercise guidance.

HESI Exit Question: A 70-year-old patient is admitted with a history of falls. Which nursing intervention should be prioritized during the admission assessment for fall risk?

  • A) Review the patient’s past medical history
  • B) Assess cognitive function and orientation
  • C) Measure blood pressure and heart rate
  • D) Inspect the skin for any breakdown

Correct Answer: B) Assess cognitive function and orientation

Explanation: Assessing cognitive function and orientation is a crucial step in fall risk assessment. Cognitive impairment can significantly increase the risk of falls due to poor judgment and awareness. While reviewing past medical history and conducting other assessments are important, assessing cognitive function takes precedence in fall prevention.

HESI Exit Question: A 78-year-old patient with osteoporosis is at risk for falls. What nursing intervention can help prevent falls related to bone health?

  • A) Encourage weight-bearing exercises
  • B) Prescribe calcium supplements
  • C) Limit fluid intake to prevent urinary incontinence
  • D) Use restraints to secure the patient in bed

Correct Answer: A) Encourage weight-bearing exercises

Explanation: Encouraging weight-bearing exercises helps improve bone density and strength, reducing the risk of fractures and falls in patients with osteoporosis. Prescribing calcium supplements can be part of the treatment plan, but exercise plays a critical role in fall prevention. Limiting fluid intake and using restraints are not appropriate interventions for fall prevention.

HESI Exit Question: A 30-year-old patient sustains a lower leg injury in a car accident and complains of severe pain, swelling, and numbness in the affected limb. Which nursing action is essential in the assessment and prevention of compartment syndrome?

  • A) Elevate the leg to reduce swelling
  • B) Administer opioid pain medication
  • C) Apply ice to the injured area
  • D) Monitor neurovascular signs and symptoms

Correct Answer: D) Monitor neurovascular signs and symptoms

Explanation: Monitoring neurovascular signs and symptoms, such as pain, sensation, color, and pulse, is crucial in assessing for compartment syndrome. Elevating the leg and administering pain medication may be appropriate interventions but should not delay ongoing assessment. Applying ice is contraindicated in compartment syndrome as it can worsen vasoconstriction and tissue damage.

A 25-year-old athlete experiences muscle swelling and pain in the calf after an intense workout. Which nursing action is important in the prevention of exercise-induced compartment syndrome?

  • A) Advise the patient to continue exercising regularly
  • B) Apply a compression bandage to the calf
  • C) Instruct the patient on proper warm-up and cool-down exercises
  • D) Administer non-steroidal anti-inflammatory drugs (NSAIDs)

Correct Answer: C) Instruct the patient on proper warm-up and cool-down exercises

Explanation: Proper warm-up and cool-down exercises can help prevent exercise-induced compartment syndrome by gradually preparing and relaxing the muscles. Continuing regular exercise is important but must be done safely. Applying a compression bandage and administering NSAIDs may not be sufficient for prevention.

HESI Exit Question: A 40-year-old patient undergoes surgery for compartment syndrome in the forearm. What is a key nursing consideration in postoperative care for this patient?

  • A) Encourage early mobilization of the affected arm
  • B) Apply ice to reduce swelling
  • C) Assess for signs of wound infection
  • D) Administer opioid pain medication as needed

Correct Answer: C) Assess for signs of wound infection

Explanation: Postoperative assessment for wound infection is critical in the care of a patient with compartment syndrome who has undergone surgery. Encouraging early mobilization, applying ice, and administering pain medication are important but should be done in conjunction with wound assessment and infection prevention.

HESI Exit Question: A 22-year-old patient presents with a suspected compartment syndrome in the thigh after a sports injury. What immediate nursing action is crucial in this situation?

  • A) Elevate the affected thigh
  • B) Administer an anti-inflammatory medication
  • C) Remove any constrictive dressings or garments
  • D) Encourage the patient to perform stretching exercises

Correct Answer: C) Remove any constrictive dressings or garments

Explanation: Removing constrictive dressings or garments is crucial in relieving pressure and preventing further damage in a suspected compartment syndrome. Elevation, administering medication, and stretching exercises should not be done until the pressure is relieved.

HESI Exit Question: A 55-year-old patient with diabetes develops compartment syndrome in the foot after a minor injury. What nursing intervention is essential in the care of this patient to prevent complications?

  • A) Encourage aggressive range of motion exercises
  • B) Monitor blood glucose levels closely
  • C) Administer heparin for anticoagulation
  • D) Apply a warm compress to the affected foot

Correct Answer: B) Monitor blood glucose levels closely

Explanation: Close monitoring of blood glucose levels is essential in diabetic patients with compartment syndrome to prevent complications such as impaired wound healing and infection. Range of motion exercises should be avoided until the condition is managed. Heparin and warm compresses are not appropriate interventions for compartment syndrome.

HESI Exit Question: A 12-year-old patient with a recent diagnosis of type 1 diabetes presents with excessive thirst, frequent urination, and weight loss. The nurse measures the patient’s blood glucose level, which reads 320 mg/dL. What is the most appropriate initial nursing action for this patient?

  • A) Administer insulin as prescribed
  • B) Encourage the patient to drink water
  • C) Assess for ketones in the urine
  • D) Provide a carbohydrate-rich snack

Correct Answer: A) Administer insulin as prescribed

Explanation: In this scenario, the patient’s high blood glucose level and symptoms suggest hyperglycemia, and the initial nursing action is to administer insulin as prescribed to lower the blood glucose level. Encouraging water intake is important but should be done in conjunction with insulin administration. Assessing for ketones and providing a carbohydrate-rich snack are secondary considerations after addressing hyperglycemia.

HESI Exit Question: A 15-year-old patient with type 1 diabetes is admitted with diabetic ketoacidosis (DKA). The nurse observes the patient’s deep and rapid breathing. What is the primary nursing intervention for this patient?

  • A) Administer intravenous (IV) insulin
  • B) Encourage oral glucose intake
  • C) Administer IV fluids with electrolytes
  • D) Assess blood pressure and heart rate

Correct Answer: C) Administer IV fluids with electrolytes

Explanation: The primary intervention for a patient with DKA exhibiting deep and rapid breathing is to administer IV fluids with electrolytes to correct dehydration and electrolyte imbalances. IV insulin will be administered as well, but fluid resuscitation takes precedence. Encouraging oral glucose intake is not appropriate in DKA, and assessing blood pressure and heart rate is important but secondary to fluid administration.

HESI Exit Question: A 25-year-old patient with type 1 diabetes reports feeling shaky, sweaty, and anxious. The nurse measures the patient’s blood glucose level, which reads 50 mg/dL. What is the immediate nursing action for this patient?

  • A) Administer a rapid-acting insulin injection
  • B) Offer a carbohydrate-containing snack or drink
  • C) Assess for signs of hyperglycemia
  • D) Encourage increased physical activity

Correct Answer: B) Offer a carbohydrate-containing snack or drink

Explanation: The patient’s low blood glucose level (hypoglycemia) requires immediate intervention with a carbohydrate-containing snack or drink to raise blood sugar levels. Administering insulin or assessing for hyperglycemia would exacerbate the hypoglycemia. Encouraging physical activity is not appropriate in this situation.

HESI Exit Question: A 30-year-old patient with type 1 diabetes has a hemoglobin A1c (HbA1c) level of 9.2%. The nurse provides diabetes education and discusses the importance of glycemic control. What information should the nurse emphasize to the patient?

  • A) The need to eliminate all carbohydrate intake
  • B) The importance of regular exercise alone in managing blood glucose
  • C) The significance of monitoring blood glucose levels frequently
  • D) The potential benefits of skipping insulin doses

Correct Answer: C) The significance of monitoring blood glucose levels frequently

Explanation: The nurse should emphasize the importance of regularly monitoring blood glucose levels to track glycemic control. This allows the patient to make informed decisions about diet, exercise, and insulin management. Eliminating all carbohydrate intake, relying solely on exercise, and skipping insulin doses are not safe or effective strategies for managing diabetes.

HESI Exit Question: A 40-year-old patient with type 1 diabetes is planning to travel overseas. The patient asks the nurse about managing insulin during the trip. What advice should the nurse provide?

  • A) Skip insulin doses to avoid hypoglycemia
  • B) Adjust insulin doses based on the time zone change
  • C) Administer insulin only when returning to the home country
  • D) Keep insulin at room temperature during travel

Correct Answer: B) Adjust insulin doses based on the time zone change

Explanation: The nurse should advise the patient to adjust insulin doses based on the time zone change to maintain glycemic control while traveling. Skipping insulin doses can lead to hyperglycemia, and insulin should not be administered only upon return. Insulin should be stored appropriately, usually at room temperature, during travel.

HESI Exit Question: A 55-year-old patient with type 1 diabetes is scheduled for a surgical procedure. The patient asks whether to take insulin on the morning of surgery. What should the nurse advise?

  • A) Skip insulin on the morning of surgery
  • B) Administer insulin as usual
  • C) Take half the usual insulin dose
  • D) Discontinue insulin for 24 hours before surgery

Correct Answer: B) Administer insulin as usual

Explanation: The patient should be advised to administer insulin as usual on the morning of surgery. Skipping insulin or making significant dosage changes without medical guidance can lead to complications. The healthcare team will provide instructions for insulin management on the day of surgery.

A 28-year-old patient with type 1 diabetes is experiencing frequent episodes of hypoglycemia. What education should the nurse provide to help prevent hypoglycemic events?

  • A) Increase carbohydrate intake at each meal
  • B) Administer insulin in larger doses
  • C) Monitor blood glucose levels less frequently
  • D) Balance carbohydrate intake with insulin doses

Correct Answer: D) Balance carbohydrate intake with insulin doses

Explanation: To prevent hypoglycemia, the nurse should emphasize the importance of balancing carbohydrate intake with insulin doses. This involves calculating insulin-to-carbohydrate ratios and adjusting insulin doses accordingly. Increasing carbohydrate intake or administering larger insulin doses without consideration can worsen hypoglycemia. Monitoring blood glucose levels regularly is essential for managing diabetes effectively.

HESI Exit Question: A 32-year-old patient with type 1 diabetes is planning to engage in strenuous physical activity. What advice should the nurse provide regarding insulin management during exercise?

  • A) Skip insulin doses before exercise
  • B) Administer extra insulin before exercise
  • C) Monitor blood glucose levels before, during, and after exercise
  • D) Reduce carbohydrate intake before exercise

Correct Answer: C) Monitor blood glucose levels before, during, and after exercise

Explanation: The patient should be advised to monitor blood glucose levels before, during, and after strenuous exercise to ensure glycemic control. Skipping insulin or administering extra insulin without proper monitoring can lead to blood glucose fluctuations. Reducing carbohydrate intake before exercise may not provide sufficient energy for the activity.

HESI Exit Question: A 45-year-old patient with type 1 diabetes is considering becoming pregnant. What is a critical nursing intervention to discuss with the patient regarding diabetes management during pregnancy?

  • A) Stopping all insulin treatment during pregnancy
  • B) Avoiding prenatal vitamins and supplements
  • C) Monitoring blood glucose levels and adjusting insulin as needed
  • D) Focusing solely on postpartum diabetes care

Correct Answer: C) Monitoring blood glucose levels and adjusting insulin as needed

Explanation: During pregnancy, it’s crucial for patients with type 1 diabetes to monitor blood glucose levels closely and adjust insulin doses as needed. This is to maintain glycemic control for the health of both the mother and the developing fetus. Stopping insulin or neglecting prenatal care is not appropriate and can lead to complications.

HESI Exit Question: A 50-year-old patient with type 1 diabetes has concerns about long-term complications. What should the nurse emphasize when discussing diabetes management to reduce the risk of complications?

  • A) Strictly adhering to a high-sugar diet
  • B) Avoiding routine healthcare check-ups
  • C) Monitoring and controlling blood glucose levels consistently
  • D) Skipping insulin injections to minimize medication use

Correct Answer: C) Monitoring and controlling blood glucose levels consistently

Explanation: To reduce the risk of long-term complications in type 1 diabetes, the nurse should emphasize the importance of monitoring and controlling blood glucose levels consistently. Strict adherence to a high-sugar diet, avoiding healthcare check-ups, and skipping insulin injections are counterproductive and can lead to complications.

HESI Exit Question: A 55-year-old patient with a history of obesity and hypertension is newly diagnosed with Type 2 Diabetes. The patient is prescribed an oral antidiabetic medication, metformin. What should the nurse explain to the patient about the mechanism of action of metformin?

  • A) Metformin increases insulin production in the pancreas.
  • B) Metformin decreases insulin resistance in the body’s tissues.
  • C) Metformin stimulates the release of glucose from the liver.
  • D) Metformin enhances glucose uptake in muscle cells.

Correct Answer: B) Metformin decreases insulin resistance in the body’s tissues.

Explanation: Metformin is an oral antidiabetic medication that works by decreasing insulin resistance in the body’s tissues, allowing cells to better respond to insulin. It does not increase insulin production, stimulate glucose release from the liver, or directly enhance glucose uptake in muscle cells.

HESI Exit Question: A 62-year-old patient with Type 2 Diabetes has an HbA1c level of 8.5%. The patient is prescribed a combination therapy of metformin and a sulfonylurea. What is the primary action of sulfonylureas in diabetes management?

  • A) Increasing insulin sensitivity
  • B) Reducing glucose production by the liver
  • C) Stimulating insulin secretion from the pancreas
  • D) Inhibiting glucose absorption in the intestines

Correct Answer: C) Stimulating insulin secretion from the pancreas.

Explanation: Sulfonylureas stimulate insulin secretion from the pancreas, increasing the availability of insulin. This helps lower blood glucose levels. They do not directly increase insulin sensitivity, reduce glucose production by the liver, or inhibit glucose absorption in the intestines.

HESI Exit Question: A 48-year-old patient with Type 2 Diabetes is experiencing episodes of hypoglycemia. The patient is taking metformin and a sulfonylurea. What education should the nurse provide to prevent hypoglycemic events?

  • A) Increase the dose of metformin
  • B) Avoid regular meals and snacks
  • C) Monitor blood glucose levels frequently
  • D) Skip sulfonylurea doses on alternate days

Correct Answer: C) Monitor blood glucose levels frequently.

Explanation: To prevent hypoglycemia, the nurse should educate the patient to monitor blood glucose levels frequently, especially when taking medications like sulfonylureas that can lower blood sugar. Increasing metformin dose, avoiding meals and snacks, or skipping sulfonylurea doses are not appropriate strategies and can worsen the condition.

HESI Exit Question: A 50-year-old patient with Type 2 Diabetes is prescribed insulin therapy in addition to oral antidiabetic medications. The patient asks why insulin is necessary. How should the nurse explain the role of insulin in diabetes management?

  • A) “Insulin increases blood sugar levels for energy.”
  • B) “Insulin helps the body use glucose for energy.”
  • C) “Insulin reduces the need for oral medications.”
  • D) “Insulin prevents the release of glucose from the liver.”

Correct Answer: B) “Insulin helps the body use glucose for energy.”

Explanation: Insulin facilitates the uptake of glucose by cells, allowing the body to use glucose for energy. It does not increase blood sugar levels, reduce the need for oral medications, or prevent the release of glucose from the liver.

HESI Exit Question: A 58-year-old patient with Type 2 Diabetes is undergoing a surgical procedure. The patient is on insulin therapy. What guidance should the nurse provide regarding insulin management on the day of surgery?

  • A) Skip all insulin doses on the day of surgery.
  • B) Administer insulin as usual on the day of surgery.
  • C) Reduce insulin doses by half on the day of surgery.
  • D) Discontinue insulin therapy for 24 hours before surgery.

Correct Answer: B) Administer insulin as usual on the day of surgery.

Explanation: The patient should be advised to administer insulin as usual on the day of surgery, as guided by the healthcare team. Skipping insulin or making significant dosage changes without medical guidance can lead to complications. The healthcare team will provide instructions for insulin management on the day of surgery.

HESI Exit Question: A 60-year-old patient with Type 2 Diabetes is concerned about managing blood glucose levels during illness. The patient asks the nurse for advice when feeling unwell. What guidance should the nurse provide regarding diabetes management during illness?

  • A) Stop taking all diabetes medications during illness.
  • B) Increase the dose of insulin when feeling unwell.
  • C) Monitor blood glucose levels more frequently.
  • D) Consume sugary foods and drinks to maintain energy.

Correct Answer: C) Monitor blood glucose levels more frequently.

Explanation: During illness, it’s important for patients with Type 2 Diabetes to monitor blood glucose levels more frequently to ensure they remain within the target range. Stopping medications, increasing insulin without guidance, or consuming sugary foods and drinks can lead to worsening of blood sugar control.

HESI Exit Question: A 45-year-old patient with Type 2 Diabetes is overweight and struggling to manage blood glucose levels. The patient is interested in weight loss strategies. What should the nurse recommend for effective weight management in diabetes?

  • A) Crash dieting for rapid weight loss.
  • B) Skipping meals to reduce calorie intake.
  • C) Consistent portion control and regular physical activity.
  • D) Frequent snacking throughout the day.

Correct Answer: C) Consistent portion control and regular physical activity.

Explanation: Effective weight management in diabetes involves consistent portion control and regular physical activity. Crash dieting, skipping meals, or frequent snacking are not recommended strategies and can negatively impact blood sugar control.

HESI Exit Question: A 70-year-old patient with Type 2 Diabetes is experiencing peripheral neuropathy. The patient asks the nurse for advice on managing neuropathic pain. What should the nurse suggest?

  • A) Apply heat directly to the affected area.
  • B) Take over-the-counter painkillers regularly.
  • C) Maintain strict blood sugar control.
  • D) Avoid all physical activity to reduce pain.

Correct Answer: C) Maintain strict blood sugar control.

Explanation: To manage neuropathic pain in diabetes, it’s important to maintain strict blood sugar control. This can help prevent or slow the progression of neuropathy. Applying heat may provide temporary relief, but it’s not a long-term solution. Taking painkillers regularly should be done under medical guidance. Avoiding physical activity is not recommended, as it can have long-term negative effects.

HESI Exit Question: A 55-year-old patient with Type 2 Diabetes is prescribed a glucagon-like peptide-1 (GLP-1) receptor agonist. What should the nurse educate the patient about regarding the use of GLP-1 receptor agonists?

  • A) GLP-1 receptor agonists can cause hypoglycemia.
  • B) GLP-1 receptor agonists should be taken with meals.
  • C) GLP-1 receptor agonists are insulin replacements.
  • D) GLP-1 receptor agonists should be avoided in obesity.

Correct Answer: B) GLP-1 receptor agonists should be taken with meals.

Explanation: GLP-1 receptor agonists should be taken with meals to help regulate blood sugar levels. They are less likely to cause hypoglycemia compared to some other medications. GLP-1 receptor agonists are not insulin replacements, and they can be used in obesity management.

HESI Exit Question: A 65-year-old patient with Type 2 Diabetes is undergoing a cardiac evaluation. The patient asks the nurse about the link between diabetes and cardiovascular disease. How should the nurse explain this relationship?

  • A) “Diabetes has no impact on cardiovascular health.”
  • B) “Diabetes increases the risk of cardiovascular disease.”
  • C) “Cardiovascular disease has no connection to diabetes.”
  • D) “Diabetes only affects the lungs.”

Correct Answer: B) “Diabetes increases the risk of cardiovascular disease.”

Explanation: Diabetes is a significant risk factor for cardiovascular disease. Elevated blood sugar levels can lead to damage to blood vessels and increase the risk of heart-related complications. It’s important for individuals with diabetes to manage their condition and maintain cardiovascular health through a healthy lifestyle and medical management.

HESI Exit Question: A 25-year-old patient with a known history of Type 1 Diabetes presents to the emergency department with symptoms of DKA, including polyuria, polydipsia, and fruity breath odor. Vital signs reveal tachycardia and hypotension. What is the most critical initial nursing intervention for this patient?

  • A) Administer insulin
  • B) Provide intravenous (IV) fluids
  • C) Offer a carbohydrate-rich snack
  • D) Administer a diuretic

Correct Answer: B) Provide intravenous (IV) fluids

Explanation: In the management of DKA, the most critical initial nursing intervention is to provide IV fluids to correct dehydration and restore hemodynamic stability. Insulin therapy is essential but should follow fluid resuscitation. Administering a diuretic or offering a carbohydrate-rich snack is contraindicated in the acute phase of DKA.

HESI Exit Question: HESI Exit Question: A 35-year-old patient with Type 1 Diabetes is admitted with DKA. Laboratory results show a blood glucose level of 520 mg/dL, arterial pH of 7.20, and bicarbonate (HCO3-) level of 15 mEq/L. The patient is started on insulin therapy. What should the nurse monitor closely to assess the patient’s response to treatment?

  • A) Serum potassium levels
  • B) Arterial oxygen saturation
  • C) Serum calcium levels
  • D) Hemoglobin A1c levels

Correct Answer: A) Serum potassium levels

Explanation: In the management of DKA, the nurse should closely monitor serum potassium levels, as insulin administration can lead to shifts in potassium levels. Hypokalemia is a potential complication that must be avoided. Monitoring arterial oxygen saturation, serum calcium levels, and hemoglobin A1c levels is important in other clinical contexts but not specific to DKA management.

HESI Exit Question: A 40-year-old patient with Type 2 Diabetes presents with symptoms of DKA, including nausea, vomiting, and confusion. The patient’s blood glucose level is 650 mg/dL, and laboratory results indicate metabolic acidosis. What additional assessment should the nurse prioritize?

  • A) Pain assessment
  • B) Neurologic assessment
  • C) Cardiac assessment
  • D) Gastrointestinal assessment

Correct Answer: B) Neurologic assessment

Explanation: In a patient with DKA, neurologic assessment is crucial as severe hyperglycemia and metabolic acidosis can lead to altered mental status, including confusion, lethargy, or coma. The nurse should monitor the patient’s neurological status closely. While other assessments may be important, prioritizing neurologic assessment is essential in this context.

HESI Exit Question: A 30-year-old patient with Type 1 Diabetes is receiving insulin therapy for DKA. The patient’s blood glucose levels are gradually decreasing, and the acidosis is improving. What nursing intervention is essential to prevent hypoglycemia in this patient?

  • A) Administering a continuous insulin infusion
  • B) Administering a bolus of rapid-acting insulin
  • C) Withholding insulin for several hours
  • D) Providing a high-carbohydrate meal

Correct Answer: A) Administering a continuous insulin infusion

Explanation: To prevent hypoglycemia while treating DKA, the nurse should administer a continuous insulin infusion, which allows for precise control of insulin dosing. This approach reduces the risk of rapid drops in blood glucose levels. Administering a bolus of rapid-acting insulin, withholding insulin, or providing a high-carbohydrate meal can lead to unpredictable blood glucose fluctuations.

HESI Exit Question: A 50-year-old patient with Type 2 Diabetes is admitted with DKA. The patient is started on insulin therapy and IV fluids. What information should the nurse provide to the patient about preventing future episodes of DKA?

  • A) “Avoid all carbohydrate intake.”
  • B) “Skip insulin doses to minimize medication use.”
  • C) “Monitor blood glucose levels occasionally.”
  • D) “Adhere to your prescribed insulin regimen and monitor blood glucose regularly.”

Correct Answer: D) “Adhere to your prescribed insulin regimen and monitor blood glucose regularly.”

Explanation: To prevent future episodes of DKA, the nurse should educate the patient to adhere to their prescribed insulin regimen and monitor blood glucose levels regularly. Skipping insulin doses or avoiding carbohydrates can worsen glycemic control. Occasional blood glucose monitoring is insufficient for managing diabetes effectively.

HESI Exit Question: A 28-year-old patient with Type 1 Diabetes is admitted with DKA and has been started on insulin therapy. The patient’s blood glucose levels are improving, and the acidosis is resolving. What key nursing assessment should be performed to evaluate the patient’s response to treatment?

  • A) Monitoring urine output
  • B) Assessing lung sounds
  • C) Checking skin turgor
  • D) Evaluating bowel sounds

Correct Answer: A) Monitoring urine output

Explanation: Monitoring urine output is a key assessment to evaluate the patient’s response to treatment for DKA. Increased urine output is a sign of improved hydration and resolution of metabolic acidosis. While other assessments like lung sounds, skin turgor, and bowel sounds are important, urine output is particularly relevant in this context.

HESI Exit Question: A 45-year-old patient with Type 1 Diabetes has a history of recurrent DKA episodes. The patient is seeking guidance on preventing future episodes. What should the nurse prioritize in patient education?

  • A) Reducing insulin dosage
  • B) Skipping meals to control blood sugar
  • C) Monitoring blood glucose levels consistently
  • D) Avoiding fluid intake during illness

Correct Answer: C) Monitoring blood glucose levels consistently

Explanation: To prevent future DKA episodes, the nurse should prioritize patient education on monitoring blood glucose levels consistently. Reducing insulin dosage or skipping meals can worsen glycemic control and lead to DKA. Encouraging adequate fluid intake during illness is also important to prevent dehydration and DKA.

HESI Exit Question: A 55-year-old patient with Type 2 Diabetes presents with signs of DKA. The patient is on oral antidiabetic medications. What is the most appropriate initial nursing action for this patient?

  • A) Administering a rapid-acting insulin injection
  • B) Checking fingerstick blood glucose levels
  • C) Initiating IV fluid replacement
  • D) Providing a carbohydrate-rich meal

Correct Answer: C) Initiating IV fluid replacement

Explanation: In a patient with signs of DKA, the most appropriate initial nursing action is to initiate IV fluid replacement to correct dehydration. While insulin therapy is important, fluid resuscitation takes precedence. Checking blood glucose levels and providing a carbohydrate-rich meal should follow fluid replacement.

HESI Exit Question: A 60-year-old patient with Type 1 Diabetes is recovering from an episode of DKA. The patient asks about long-term complications of diabetes. What should the nurse emphasize regarding the prevention of complications?

  • A) “There are no complications associated with diabetes.”
  • B) “Avoid insulin therapy to prevent complications.”
  • C) “Monitor blood glucose levels regularly and follow prescribed treatment plans.”
  • D) “Consume a high-sugar diet for energy.”

Correct Answer: C) “Monitor blood glucose levels regularly and follow prescribed treatment plans.”

Explanation: To prevent long-term complications of diabetes, the nurse should emphasize the importance of monitoring blood glucose levels regularly and following prescribed treatment plans. Managing diabetes with insulin therapy and a balanced diet is crucial. The statements suggesting no complications or avoiding insulin are not accurate or safe.

HESI Exit Question: A 70-year-old patient with Type 2 Diabetes is recovering from an episode of DKA. The patient expresses concerns about future episodes. What should the nurse include in the discharge teaching plan to minimize the risk of recurrence?

  • A) “Skip insulin doses when feeling unwell.”
  • B) “Avoid regular meals to control blood sugar.”
  • C) “Monitor blood glucose levels frequently and follow your prescribed insulin regimen.”
  • D) “Consume sugary snacks for energy.”

Correct Answer: C) “Monitor blood glucose levels frequently and follow your prescribed insulin regimen.”

Explanation: To minimize the risk of recurrence of DKA, the nurse should include in the discharge teaching plan the importance of monitoring blood glucose levels frequently and following the prescribed insulin regimen. Skipping insulin, avoiding regular meals, or consuming sugary snacks are not recommended strategies and can lead to complications.

HESI Exit Question: A 65-year-old patient with Type 2 Diabetes is admitted to the hospital with altered mental status, dehydration, and extreme hyperglycemia (blood glucose level of 800 mg/dL). The patient’s serum osmolality is markedly elevated. What is the most critical initial nursing intervention for this patient?

  • A) Administering a rapid-acting insulin injection
  • B) Providing oral rehydration solutions
  • C) Initiating intravenous (IV) fluid replacement
  • D) Administering a diuretic

Correct Answer: C) Initiating intravenous (IV) fluid replacement

Explanation: In the management of Hyperosmolar Hyperglycemic State (HHS), the most critical initial nursing intervention is to initiate IV fluid replacement to correct dehydration and restore hemodynamic stability. While insulin therapy is essential, fluid resuscitation takes precedence. Administering a diuretic or providing oral rehydration solutions is contraindicated in the acute phase of HHS.

HESI Exit Question: A 70-year-old patient with Type 2 Diabetes presents to the emergency department with signs of HHS, including extreme thirst and dry mucous membranes. Laboratory results show a serum osmolality of 350 mOsm/kg. What assessment should the nurse prioritize for this patient?

  • A) Neurologic assessment
  • B) Cardiac assessment
  • C) Respiratory assessment
  • D) Gastrointestinal assessment

Correct Answer: A) Neurologic assessment

Explanation: In a patient with HHS, neurologic assessment is crucial as severe hyperosmolarity can lead to neurological symptoms such as altered mental status and seizures. The nurse should monitor the patient’s neurological status closely. While other assessments are important, neurologic assessment takes precedence in this context.

HESI Exit Question: A 45-year-old patient with Type 2 Diabetes is recovering from an episode of HHS. The patient asks about preventing future episodes. What should the nurse emphasize regarding prevention?

  • A) “Reduce insulin dosage to prevent recurrence.”
  • B) “Avoid all carbohydrates in your diet.”
  • C) “Monitor blood glucose levels consistently and follow your prescribed insulin regimen.”
  • D) “Skip insulin doses when feeling unwell.”

Correct Answer: C) “Monitor blood glucose levels consistently and follow your prescribed insulin regimen.”

Explanation: To prevent future episodes of HHS, the nurse should emphasize the importance of monitoring blood glucose levels consistently and following the prescribed insulin regimen. Reducing insulin dosage, avoiding all carbohydrates, or skipping insulin doses are not recommended strategies and can lead to recurrence.

HESI Exit Question: A 55-year-old patient with Type 2 Diabetes is admitted with HHS. The patient’s blood glucose levels are gradually decreasing, and the serum osmolality is improving. What nursing intervention is essential to prevent complications in this patient?

  • A) Administering a continuous insulin infusion
  • B) Administering a bolus of rapid-acting insulin
  • C) Providing a carbohydrate-rich meal
  • D) Monitoring urine output

Correct Answer: A) Administering a continuous insulin infusion

Explanation: To prevent complications while treating HHS, the nurse should administer a continuous insulin infusion, which allows for precise control of insulin dosing. This approach reduces the risk of rapid drops in blood glucose levels. Administering a bolus of rapid-acting insulin, providing a carbohydrate-rich meal, or monitoring urine output are important but not the most essential intervention in this context.

HESI Exit Question: A 60-year-old patient with Type 1 Diabetes is recovering from an episode of HHS. The patient asks about long-term complications of diabetes. What should the nurse emphasize regarding the prevention of complications?

  • A) “There are no complications associated with diabetes.”
  • B) “Avoid insulin therapy to prevent complications.”
  • C) “Monitor blood glucose levels regularly and follow prescribed treatment plans.”
  • D) “Consume a high-sugar diet for energy.”

Correct Answer: C) “Monitor blood glucose levels regularly and follow prescribed treatment plans.”

Explanation: To prevent long-term complications of diabetes, the nurse should emphasize the importance of monitoring blood glucose levels regularly and following prescribed treatment plans. Managing diabetes with insulin therapy and a balanced diet is crucial. The statements suggesting no complications, avoiding insulin, or consuming a high-sugar diet are not accurate or safe.

HESI Exit Question: A 65-year-old patient with Type 2 Diabetes is admitted to the hospital with altered mental status, dehydration, and extreme hyperglycemia (blood glucose level of 800 mg/dL). The patient’s serum osmolality is markedly elevated. What is the most critical initial nursing intervention for this patient?

  • A) Administering a rapid-acting insulin injection
  • B) Providing oral rehydration solutions
  • C) Initiating intravenous (IV) fluid replacement
  • D) Administering a diuretic

Correct Answer: C) Initiating intravenous (IV) fluid replacement

Explanation: In the management of Hyperosmolar Hyperglycemic State (HHS), the most critical initial nursing intervention is to initiate IV fluid replacement to correct dehydration and restore hemodynamic stability. While insulin therapy is essential, fluid resuscitation takes precedence. Administering a diuretic or providing oral rehydration solutions is contraindicated in the acute phase of HHS.

HESI Exit Question: A 70-year-old patient with Type 2 Diabetes presents to the emergency department with signs of HHS, including extreme thirst and dry mucous membranes. Laboratory results show a serum osmolality of 350 mOsm/kg. What assessment should the nurse prioritize for this patient?

  • A) Neurologic assessment
  • B) Cardiac assessment
  • C) Respiratory assessment
  • D) Gastrointestinal assessment

Correct Answer: A) Neurologic assessment

Explanation: In a patient with HHS, neurologic assessment is crucial as severe hyperosmolarity can lead to neurological symptoms such as altered mental status and seizures. The nurse should monitor the patient’s neurological status closely. While other assessments are important, neurologic assessment takes precedence in this context.

HESI Exit Question: A 45-year-old patient with Type 2 Diabetes is recovering from an episode of HHS. The patient asks about preventing future episodes. What should the nurse emphasize regarding prevention?

  • A) “Reduce insulin dosage to prevent recurrence.”
  • B) “Avoid all carbohydrates in your diet.”
  • C) “Monitor blood glucose levels consistently and follow your prescribed insulin regimen.”
  • D) “Skip insulin doses when feeling unwell.”

Correct Answer: C) “Monitor blood glucose levels consistently and follow your prescribed insulin regimen.”

Explanation: To prevent future episodes of HHS, the nurse should emphasize the importance of monitoring blood glucose levels consistently and following the prescribed insulin regimen. Reducing insulin dosage, avoiding all carbohydrates, or skipping insulin doses are not recommended strategies and can lead to recurrence.

HESI Exit Question: A 55-year-old patient with Type 2 Diabetes is admitted with HHS. The patient’s blood glucose levels are gradually decreasing, and the serum osmolality is improving. What nursing intervention is essential to prevent complications in this patient?

  • A) Administering a continuous insulin infusion
  • B) Administering a bolus of rapid-acting insulin
  • C) Providing a carbohydrate-rich meal
  • D) Monitoring urine output

Correct Answer: A) Administering a continuous insulin infusion

Explanation: To prevent complications while treating HHS, the nurse should administer a continuous insulin infusion, which allows for precise control of insulin dosing. This approach reduces the risk of rapid drops in blood glucose levels. Administering a bolus of rapid-acting insulin, providing a carbohydrate-rich meal, or monitoring urine output are important but not the most essential intervention in this context.

HESI Exit Question: A 60-year-old patient with Type 1 Diabetes is recovering from an episode of HHS. The patient asks about long-term complications of diabetes. What should the nurse emphasize regarding the prevention of complications?

  • A) “There are no complications associated with diabetes.”
  • B) “Avoid insulin therapy to prevent complications.”
  • C) “Monitor blood glucose levels regularly and follow prescribed treatment plans.”
  • D) “Consume a high-sugar diet for energy.”

Correct Answer: C) “Monitor blood glucose levels regularly and follow prescribed treatment plans.”

Explanation: To prevent long-term complications of diabetes, the nurse should emphasize the importance of monitoring blood glucose levels regularly and following prescribed treatment plans. Managing diabetes with insulin therapy and a balanced diet is crucial. The statements suggesting no complications, avoiding insulin, or consuming a high-sugar diet are not accurate or safe.

HESI Exit Question: A 45-year-old patient with Type 1 Diabetes is prescribed insulin therapy. The patient is concerned about weight gain. Which type of insulin is most likely to cause weight gain?

  • A) Regular insulin (R)
  • B) NPH insulin
  • C) Lispro insulin (Humalog)
  • D) Glargine insulin (Lantus)

Correct Answer: B) NPH insulin

Explanation: NPH insulin, also known as intermediate-acting insulin, has a higher likelihood of causing weight gain compared to rapid-acting insulins like Lispro (Humalog) or long-acting insulins like Glargine (Lantus). Patients should be educated on the potential for weight gain and encouraged to maintain a balanced diet and exercise routine.

HESI Exit Question: A 55-year-old patient with Type 2 Diabetes is prescribed insulin therapy to manage blood glucose levels. The patient is worried about hypoglycemia. Which type of insulin is associated with the highest risk of hypoglycemia?

  • A) Regular insulin (R)
  • B) NPH insulin
  • C) Lispro insulin (Humalog)
  • D) Glargine insulin (Lantus)

Correct Answer: A) Regular insulin (R)

Explanation: Regular insulin (R) has a higher risk of causing hypoglycemia compared to other types of insulin. It has a longer duration of action and requires careful monitoring and meal planning to prevent low blood glucose levels. Patients should be educated about recognizing and managing hypoglycemia when using regular insulin.

HESI Exit Question: A 30-year-old patient with Type 1 Diabetes is prescribed a combination of insulin Lispro (Humalog) and Glargine (Lantus). The patient asks about the purpose of using both types of insulin. What should the nurse explain?

  • A) Lispro (Humalog) is used for long-term blood sugar control, while Glargine (Lantus) is for mealtime coverage.
  • B) Lispro (Humalog) is for basal insulin needs, while Glargine (Lantus) is for bolus insulin needs.
  • C) Lispro (Humalog) provides rapid blood sugar control at mealtime, while Glargine (Lantus) provides 24-hour basal insulin coverage.
  • D) Lispro (Humalog) is used to correct high blood sugar levels, while Glargine (Lantus) is for bedtime use.

Correct Answer: C) Lispro (Humalog) provides rapid blood sugar control at mealtime, while Glargine (Lantus) provides 24-hour basal insulin coverage.

Explanation: Lispro (Humalog) is a rapid-acting insulin used to control blood sugar levels at mealtime, while Glargine (Lantus) is a long-acting insulin that provides a basal level of insulin throughout the day and night. This combination helps mimic the body’s natural insulin production and control blood glucose effectively.

HESI Exit Question: A 35-year-old patient with Type 2 Diabetes is prescribed insulin therapy for the first time. The patient is anxious about the injections. What should the nurse teach the patient about insulin injection sites?

  • A) “You should always inject insulin in the same spot to avoid variation.”
  • B) “Rotate injection sites within the same region to prevent lipodystrophy.”
  • C) “Inject insulin only in the abdomen for the best absorption.”
  • D) “Use the thigh for insulin injections to minimize discomfort.”

Correct Answer: B) “Rotate injection sites within the same region to prevent lipodystrophy.”

Explanation: To prevent lipodystrophy (changes in fat tissue), patients should rotate injection sites within the same region (e.g., abdomen, thigh, or arm). Using the same spot repeatedly can lead to absorption issues and tissue damage. The nurse should encourage proper site rotation and provide guidance on injection technique to ease the patient’s anxiety.

HESI Exit Question: A 50-year-old patient with Type 2 Diabetes is prescribed a combination of insulin and oral antidiabetic medications. The patient asks about the timing of insulin injections. What should the nurse advise regarding the timing of insulin?

  • A) “You should take insulin immediately after a meal for better blood sugar control.”
  • B) “Inject insulin 30 minutes before a meal to allow it to work effectively.”
  • C) “It doesn’t matter when you take insulin; you can take it at any time of the day.”
  • D) “Inject insulin only when your blood sugar levels are very high.”

Correct Answer: B) “Inject insulin 30 minutes before a meal to allow it to work effectively.”

Explanation: Injecting insulin approximately 30 minutes before a meal allows it to start working when blood sugar levels begin to rise due to food intake. This timing helps match insulin action with the body’s glucose absorption, promoting better blood sugar control. Patients should be educated on the importance of mealtime insulin dosing.

HESI Exit Question: A 60-year-old patient with Type 1 Diabetes is admitted with concerns about insulin dose adjustments during illness. What should the nurse teach the patient regarding insulin management when sick?

  • A) “Continue your regular insulin doses even if you’re not eating.”
  • B) “Reduce your insulin doses to prevent hypoglycemia.”
  • C) “Consult your healthcare provider for individualized insulin adjustments.”
  • D) “Stop taking insulin until you recover from illness.”

Correct Answer: C) “Consult your healthcare provider for individualized insulin adjustments.”

Explanation: During illness, insulin needs may vary, and individualized adjustments are essential. Patients should consult their healthcare provider for guidance on insulin management. Continuing regular doses without eating can lead to hypoglycemia, while stopping insulin can result in hyperglycemia.

HESI Exit Question: A 70-year-old patient with Type 2 Diabetes has been using insulin for several years. The patient has noticed a lump at the injection site. What should the nurse explain about this situation?

  • A) “Lumps at injection sites are common and harmless.”
  • B) “Inject insulin deeper to avoid lumps.”
  • C) “Change the insulin type to prevent lumps.”
  • D) “Avoid insulin injections altogether to prevent lumps.”

Correct Answer: A) “Lumps at injection sites are common and harmless.”

Explanation: Lumps at injection sites, known as lipohypertrophy, can occur but are typically harmless. The nurse should educate the patient about the importance of proper injection technique, including site rotation, to prevent these lumps. Changing insulin type or avoiding insulin injections is not the solution.

HESI Exit Question: A 35-year-old patient with Type 1 Diabetes is prescribed insulin Lispro (Humalog) and Glargine (Lantus). The patient inquires about the storage of insulin. What should the nurse advise regarding insulin storage?

  • A) “Store insulin in the freezer to prolong its shelf life.”
  • B) “Keep insulin at room temperature for easy access.”
  • C) “Refrigerate insulin to maintain its potency.”
  • D) “Expose insulin to direct sunlight to enhance its effectiveness.”

Correct Answer: C) “Refrigerate insulin to maintain its potency.”

Explanation: Insulin should be stored in the refrigerator to maintain its potency. However, it should not be frozen or exposed to direct sunlight. Patients should be educated about proper insulin storage to ensure its effectiveness.

HESI Exit Question: A 45-year-old patient with Type 2 Diabetes is experiencing symptoms of hypoglycemia after taking insulin. What is the most appropriate nursing intervention for this patient?

  • A) Administering a bolus of insulin to correct high blood sugar levels.
  • B) Providing a carbohydrate source, such as fruit juice or glucose gel, to raise blood sugar.
  • C) Encouraging the patient to skip the next insulin dose to prevent further hypoglycemia.
  • D) Advising the patient to reduce carbohydrate intake during meals.

Correct Answer: B) Providing a carbohydrate source, such as fruit juice or glucose gel, to raise blood sugar.

Explanation: The most appropriate nursing intervention for hypoglycemia is to provide a rapid-acting carbohydrate source to raise blood sugar levels. Administering more insulin, skipping doses, or reducing carbohydrate intake can worsen hypoglycemia.

HESI Exit Question: A 50-year-old patient with Type 2 Diabetes is prescribed insulin therapy. The patient is concerned about the cost of insulin. What should the nurse advise regarding options for affordable insulin?

  • A) “Insulin is not available at lower costs; you must pay the full price.”
  • B) “Consider discussing lower-cost insulin options with your healthcare provider or pharmacist.”
  • C) “Stop using insulin to save money.”
  • D) “Purchase insulin from unregulated online sources to save on costs.”

Correct Answer: B) “Consider discussing lower-cost insulin options with your healthcare provider or pharmacist.”

Explanation: Patients concerned about the cost of insulin should be advised to discuss lower-cost insulin options with their healthcare provider or pharmacist. Stopping insulin or purchasing from unregulated sources can pose significant health risks.

HESI Exit Question: A 55-year-old patient with Type 2 Diabetes has been prescribed metformin (Glucophage). The patient is concerned about side effects. Which of the following is a common side effect of metformin?

  • A) Hypoglycemia
  • B) Weight gain
  • C) Gastrointestinal upset
  • D) Elevated blood pressure

Correct Answer: C) Gastrointestinal upset

Explanation: Gastrointestinal upset, including diarrhea and abdominal discomfort, is a common side effect of metformin. It is important to educate the patient about these potential side effects and recommend taking the medication with food to minimize discomfort.

HESI Exit Question: A 45-year-old patient with Type 2 Diabetes is prescribed sulfonylurea medication (e.g., glyburide). What is the primary mechanism of action of sulfonylureas?

  • A) Increasing insulin sensitivity
  • B) Stimulating insulin secretion from the pancreas
  • C) Reducing glucose absorption in the intestines
  • D) Enhancing glucose utilization in muscle cells

Correct Answer: B) Stimulating insulin secretion from the pancreas

Explanation: Sulfonylureas work by stimulating the pancreas to release more insulin, increasing insulin secretion. This helps lower blood glucose levels. Patients should be aware of the risk of hypoglycemia with sulfonylurea use and the importance of regular monitoring.

HESI Exit Question: A 60-year-old patient with Type 2 Diabetes is prescribed an alpha-glucosidase inhibitor (e.g., acarbose). How does this class of oral hypoglycemics primarily work?

  • A) Increasing insulin sensitivity
  • B) Stimulating insulin secretion from the pancreas
  • C) Slowing carbohydrate digestion and absorption
  • D) Enhancing glucose uptake by cells

Correct Answer: C) Slowing carbohydrate digestion and absorption

Explanation: Alpha-glucosidase inhibitors, such as acarbose, work by slowing down the digestion and absorption of carbohydrates in the intestines. This leads to more gradual rises in blood sugar after meals. Patients should be educated about taking these medications with the first bite of each main meal.

HESI Exit Question: A 35-year-old patient with Type 2 Diabetes is prescribed a DPP-4 inhibitor (e.g., sitagliptin). What is the primary action of DPP-4 inhibitors in managing blood glucose levels?

  • A) Enhancing insulin secretion
  • B) Increasing insulin sensitivity
  • C) Inhibiting the breakdown of incretin hormones
  • D) Promoting glucose excretion in urine

Correct Answer: C) Inhibiting the breakdown of incretin hormones

Explanation: DPP-4 inhibitors, like sitagliptin, primarily work by inhibiting the breakdown of incretin hormones. Incretin hormones play a role in regulating insulin secretion and glucose control. These medications help increase insulin release and decrease glucagon secretion after meals.

HESI Exit Question: A 50-year-old patient with Type 2 Diabetes is prescribed a SGLT2 inhibitor (e.g., empagliflozin). How do SGLT2 inhibitors primarily affect blood glucose levels?

  • A) Increasing insulin production
  • B) Reducing glucose absorption in the intestines
  • C) Enhancing glucose uptake by muscle cells
  • D) Promoting glucose excretion in urine

Correct Answer: D) Promoting glucose excretion in urine

Explanation: SGLT2 inhibitors, such as empagliflozin, work by promoting the excretion of glucose in the urine. They inhibit glucose reabsorption in the kidneys, leading to decreased blood glucose levels. Patients should be advised to monitor for signs of urinary tract infections while on these medications.

HESI Exit Question: A 40-year-old patient with Type 2 Diabetes is prescribed meglitinide medication (e.g., repaglinide). How does meglitinide primarily lower blood glucose levels?

  • A) Increasing insulin sensitivity
  • B) Stimulating insulin secretion from the pancreas
  • C) Slowing carbohydrate digestion and absorption
  • D) Enhancing glucose utilization in muscle cells

Correct Answer: B) Stimulating insulin secretion from the pancreas

Explanation: Meglitinides like repaglinide work by stimulating the pancreas to release insulin, similar to sulfonylureas. They enhance insulin secretion and should be taken before meals to match their action with food intake.

HESI Exit Question: A 55-year-old patient with Type 2 Diabetes is prescribed a thiazolidinedione (e.g., pioglitazone). What is the primary mechanism of action of thiazolidinediones?

  • A) Increasing insulin secretion
  • B) Reducing glucose absorption in the intestines
  • C) Enhancing glucose uptake by muscle cells
  • D) Promoting glucose excretion in urine

Correct Answer: C) Enhancing glucose uptake by muscle cells

Explanation: Thiazolidinediones, such as pioglitazone, primarily work by enhancing glucose uptake by muscle cells. They improve insulin sensitivity in peripheral tissues. Patients should be informed about potential side effects, including fluid retention and heart-related issues.

HESI Exit Question: A 65-year-old patient with Type 2 Diabetes is prescribed a GLP-1 receptor agonist (e.g., liraglutide). What is the primary action of GLP-1 receptor agonists in managing blood glucose levels?

  • A) Increasing insulin secretion
  • B) Slowing carbohydrate digestion and absorption
  • C) Inhibiting the breakdown of incretin hormones
  • D) Enhancing glucose excretion in urine

Correct Answer: A) Increasing insulin secretion

Explanation: GLP-1 receptor agonists, like liraglutide, primarily increase insulin secretion in response to elevated blood glucose levels. They also suppress glucagon secretion, slow gastric emptying, and promote satiety. Patients should be advised on proper injection technique and potential gastrointestinal side effects.

HESI Exit Question: A 50-year-old patient with Type 2 Diabetes is prescribed a combination of metformin and a DPP-4 inhibitor (e.g., saxagliptin). What is the rationale for combining these two medications?

  • A) To enhance glucose excretion in urine
  • B) To reduce the risk of hypoglycemia
  • C) To improve insulin sensitivity
  • D) To stimulate insulin secretion

Correct Answer: B) To reduce the risk of hypoglycemia

Explanation: Combining metformin with a DPP-4 inhibitor helps reduce the risk of hypoglycemia, as DPP-4 inhibitors primarily work by increasing insulin secretion in response to elevated blood glucose levels. Metformin complements this effect by improving insulin sensitivity and reducing hepatic glucose production.

HESI Exit Question: A 55-year-old patient with Type 2 Diabetes is prescribed a sodium-glucose cotransporter-2 (SGLT2) inhibitor (e.g., canagliflozin). In addition to glucose control, what other benefit may be associated with SGLT2 inhibitors?

  • A) Increased risk of hypoglycemia
  • B) Weight gain
  • C) Cardiovascular protection
  • D) Enhanced insulin secretion

Correct Answer: C) Cardiovascular protection

Explanation: SGLT2 inhibitors have been associated with cardiovascular benefits, including reduced risk of cardiovascular events and heart failure hospitalization. They may also have potential benefits for blood pressure and weight management. Patients should be educated about these potential advantages.

HESI Exit Question: A 35-year-old patient presents with symptoms such as weight loss, palpitations, and heat intolerance. Based on these symptoms, which diagnostic test is most appropriate for assessing hyperthyroidism?

  • A) Thyroid function panel
  • B) Complete blood count (CBC)
  • C) Electrocardiogram (ECG)
  • D) Chest X-ray

Correct Answer: A) Thyroid function panel

Explanation: A thyroid function panel, including measures of thyroid hormones (T3 and T4) and thyroid-stimulating hormone (TSH), is the primary diagnostic test for hyperthyroidism. Elevated T3 and T4 levels and low TSH levels are indicative of hyperthyroidism. Additional tests may be performed to determine the underlying cause.

HESI Exit Question: A 45-year-old patient is suspected of having hyperthyroidism. Which of the following imaging studies can provide valuable information about the size and function of the thyroid gland?

  • A) Magnetic Resonance Imaging (MRI)
  • B) Computed Tomography (CT) scan
  • C) Thyroid ultrasound
  • D) Positron Emission Tomography (PET) scan

Correct Answer: C) Thyroid ultrasound

Explanation: A thyroid ultrasound is a valuable diagnostic tool for assessing the size, structure, and function of the thyroid gland. It can help identify thyroid nodules, inflammation, or enlargement associated with hyperthyroidism.

HESI Exit Question: A 50-year-old patient with suspected hyperthyroidism is scheduled for a radioactive iodine uptake (RAIU) test. What is the primary purpose of this test?

  • A) To assess thyroid gland vascularity
  • B) To detect thyroid antibodies
  • C) To measure thyroid hormone levels
  • D) To evaluate thyroid function and determine the cause of hyperthyroidism

Correct Answer: D) To evaluate thyroid function and determine the cause of hyperthyroidism

Explanation: A radioactive iodine uptake (RAIU) test is used to evaluate thyroid function and determine the underlying cause of hyperthyroidism. It measures the thyroid’s ability to take up and trap radioactive iodine, helping differentiate between different types of hyperthyroidism, such as Graves’ disease or thyroid nodules.

HESI Exit Question: A 55-year-old patient with suspected hyperthyroidism is undergoing a thyroid scan. What should the nurse educate the patient about this procedure?

  • A) The need for fasting before the scan
  • B) The importance of avoiding iodine-containing foods
  • C) The use of intravenous contrast dye during the scan
  • D) The need for a full bladder during the procedure

Correct Answer: B) The importance of avoiding iodine-containing foods

Explanation: Before a thyroid scan, patients should be educated about avoiding iodine-containing foods and contrast agents that can interfere with the test. Fasting, intravenous contrast, and bladder preparation are typically not necessary for a thyroid scan.

HESI Exit Question: A 60-year-old patient is diagnosed with hyperthyroidism, and the healthcare provider orders a thyroid-stimulating immunoglobulin (TSI) test. What does an elevated TSI level indicate?

  • A) Thyroid cancer
  • B) Thyroid inflammation
  • C) Graves’ disease
  • D) Thyroid nodules

Correct Answer: C) Graves’ disease

Explanation: An elevated thyroid-stimulating immunoglobulin (TSI) level is indicative of Graves’ disease, an autoimmune condition that causes hyperthyroidism. It is important to differentiate Graves’ disease from other causes of hyperthyroidism to determine appropriate treatment.

HESI Exit Question: A 40-year-old patient with hyperthyroidism is scheduled for a thyroid fine-needle aspiration (FNA) biopsy. What is the primary purpose of this procedure?

  • A) To measure thyroid hormone levels
  • B) To assess thyroid gland vascularity
  • C) To determine the size of thyroid nodules
  • D) To obtain a tissue sample for cytology and confirm or rule out thyroid cancer

Correct Answer: D) To obtain a tissue sample for cytology and confirm or rule out thyroid cancer

Explanation: A thyroid fine-needle aspiration (FNA) biopsy is performed to obtain a tissue sample from thyroid nodules for cytological examination. It helps confirm or rule out thyroid cancer, allowing for appropriate treatment planning.

HESI Exit Question: A 45-year-old patient with hyperthyroidism is undergoing a radioactive iodine therapy. What is the primary goal of this treatment?

  • A) To reduce thyroid hormone production
  • B) To increase thyroid gland size
  • C) To accelerate heart rate
  • D) To stimulate thyroid antibody production

Correct Answer: A) To reduce thyroid hormone production

Explanation: Radioactive iodine therapy is used to reduce thyroid hormone production in patients with hyperthyroidism. It involves the use of radioactive iodine to target and destroy overactive thyroid tissue, ultimately restoring thyroid function to normal or hypothyroid levels.

HESI Exit Question: A 55-year-old patient with hyperthyroidism is scheduled for a thyroidectomy. What preoperative teaching should the nurse provide to the patient?

  • A) Expect to be discharged the same day after surgery.
  • B) Anticipate lifelong thyroid hormone replacement therapy after surgery.
  • C) Plan to resume regular diet immediately postoperatively.
  • D) Avoid any neck movement for at least a week after surgery.

Correct Answer: B) Anticipate lifelong thyroid hormone replacement therapy after surgery.

Explanation: Patients undergoing thyroidectomy should be educated about the need for lifelong thyroid hormone replacement therapy postoperatively. This is essential to maintain normal thyroid function. Discharge timing, diet, and neck movement will depend on the individual patient’s condition and surgeon’s recommendations.

HESI Exit Question: A 60-year-old patient with hyperthyroidism is receiving treatment with antithyroid medications (e.g., methimazole). What is the purpose of these medications?

  • A) To destroy thyroid tissue
  • B) To stimulate thyroid hormone production
  • C) To inhibit excessive thyroid hormone synthesis
  • D) To promote thyroid gland enlargement

Correct Answer: C) To inhibit excessive thyroid hormone synthesis

Explanation: Antithyroid medications, such as methimazole, are used to inhibit the excessive synthesis of thyroid hormones by the thyroid gland. They help normalize thyroid function in patients with hyperthyroidism.

HESI Exit Question: A 70-year-old patient with hyperthyroidism is experiencing severe thyrotoxicosis symptoms. What immediate intervention should the nurse prioritize?

  • A) Administering antithyroid medications
  • B) Providing emotional support
  • C) Initiating radioactive iodine therapy
  • D) Managing life-threatening symptoms and stabilizing the patient’s condition

Correct Answer: D) Managing life-threatening symptoms and stabilizing the patient’s condition

Explanation: In cases of severe thyrotoxicosis, the immediate priority is to manage life-threatening symptoms such as cardiac arrhythmias, hyperthermia, and dehydration. This may involve administering medications, providing supportive care, and stabilizing the patient’s condition before pursuing definitive treatment options.

HESI Exit Question: A 40-year-old patient with hypothyroidism is prescribed levothyroxine (Synthroid). Which statement about levothyroxine administration is correct?

  • A) It should be taken with a full glass of milk.
  • B) It is best taken on an empty stomach in the morning.
  • C) It can be mixed with any food or beverage for easier intake.
  • D) It should be taken with calcium-rich foods to enhance absorption.

Correct Answer: B) It is best taken on an empty stomach in the morning.

Explanation: Levothyroxine should be taken on an empty stomach in the morning to maximize absorption. Calcium-rich foods and certain medications can interfere with absorption, so it is essential to take it separately from them.

HESI Exit Question: A 55-year-old patient with hyperthyroidism is prescribed propylthiouracil (PTU). What is the primary action of PTU in managing hyperthyroidism?

  • A) Increasing thyroid hormone production
  • B) Inhibiting thyroid hormone synthesis
  • C) Stimulating the thyroid gland
  • D) Promoting thyroid hormone uptake

Correct Answer: B) Inhibiting thyroid hormone synthesis

Explanation: PTU inhibits the synthesis of thyroid hormones in the thyroid gland. It is used to manage hyperthyroidism by reducing the production of excess thyroid hormones.

HESI Exit Question: A 50-year-old patient with hypothyroidism is prescribed liothyronine (Cytomel). What is the primary difference between liothyronine and levothyroxine?

  • A) Liothyronine is administered intravenously.
  • B) Liothyronine has a longer duration of action.
  • C) Liothyronine is the inactive form of thyroid hormone.
  • D) Liothyronine is the active form of thyroid hormone.

Correct Answer: D) Liothyronine is the active form of thyroid hormone.

Explanation: Liothyronine is the active form of thyroid hormone (T3), while levothyroxine is converted to the active form (T3) in the body. Liothyronine is used in cases where rapid correction of hypothyroidism is needed.

HESI Exit Question: A 35-year-old patient with hypothyroidism is prescribed thyroid desiccated (Armour Thyroid). What is a unique feature of thyroid desiccated compared to synthetic thyroid medications?

  • A) It is less expensive.
  • B) It has a more extended duration of action.
  • C) It contains both T3 and T4 hormones.
  • D) It has fewer side effects.

Correct Answer: C) It contains both T3 and T4 hormones.

Explanation: Thyroid desiccated contains both T3 and T4 hormones, while synthetic thyroid medications typically contain only T4 (levothyroxine). This can be advantageous for some patients who may benefit from the additional T3 hormone.

HESI Exit Question: A 45-year-old patient with hyperthyroidism is prescribed radioactive iodine therapy. What is the primary goal of this treatment?

  • A) To stimulate thyroid hormone production
  • B) To increase thyroid gland size
  • C) To destroy overactive thyroid tissue
  • D) To promote thyroid antibody production

Correct Answer: C) To destroy overactive thyroid tissue

Explanation: Radioactive iodine therapy is used to destroy overactive thyroid tissue, reducing thyroid hormone production and treating hyperthyroidism.

HESI Exit Question: A 60-year-old patient with hypothyroidism is prescribed a combination of levothyroxine and liothyronine. What is the rationale for this combination therapy?

  • A) To reduce the risk of thyroid cancer
  • B) To provide a more extended duration of action
  • C) To achieve a more balanced ratio of T3 and T4 hormones
  • D) To stimulate thyroid antibody production

Correct Answer: C) To achieve a more balanced ratio of T3 and T4 hormones

Explanation: Combining levothyroxine (T4) and liothyronine (T3) can help achieve a more balanced ratio of T3 to T4 hormones in some patients, potentially improving symptom control.

HESI Exit Question: A 45-year-old patient with hyperthyroidism is prescribed beta-blockers (e.g., propranolol) in addition to antithyroid medications. What is the primary purpose of beta-blockers in this treatment regimen?

  • A) To increase thyroid hormone production
  • B) To inhibit thyroid hormone synthesis
  • C) To manage symptoms such as tachycardia and tremors
  • D) To promote thyroid antibody production

Correct Answer: C) To manage symptoms such as tachycardia and tremors

Explanation: Beta-blockers are used in hyperthyroidism to manage symptoms such as tachycardia, palpitations, and tremors. They do not directly affect thyroid hormone levels but provide symptomatic relief.

HESI Exit Question: A 50-year-old patient with hypothyroidism is receiving thyroid replacement therapy with levothyroxine. The patient reports experiencing palpitations and insomnia. What action should the nurse take?

  • A) Increase the levothyroxine dose
  • B) Discontinue levothyroxine immediately
  • C) Educate the patient on proper medication timing
  • D) Refer the patient for radioactive iodine therapy

Correct Answer: C) Educate the patient on proper medication timing

Explanation: Palpitations and insomnia can be signs of excessive thyroid replacement. The nurse should educate the patient on the importance of taking levothyroxine on an empty stomach and avoiding medications or foods that may interfere with absorption.

HESI Exit Question: A 55-year-old patient with hyperthyroidism is prescribed radioactive iodine therapy. What precautions should the nurse provide to the patient regarding radiation exposure?

  • A) Avoid contact with others for one week
  • B) Limit exposure to sunlight
  • C) Increase iodine-rich foods in the diet
  • D) Resume regular activities immediately after treatment

Correct Answer: A) Avoid contact with others for one week

Explanation: After radioactive iodine therapy, patients should avoid close contact with others, especially pregnant women and children, for a specified period to minimize radiation exposure to others. They should also follow any additional instructions provided by the healthcare provider.

HESI Exit Question: A 65-year-old patient with hypothyroidism is prescribed thyroid replacement therapy. What is the expected outcome of successful thyroid hormone replacement in this patient?

  • A) Rapid weight loss
  • B) Resolution of fatigue and cold intolerance
  • C) Onset of hyperthyroid symptoms
  • D) Decreased heart rate

Correct Answer: B) Resolution of fatigue and cold intolerance

Explanation: Successful thyroid hormone replacement therapy in a patient with hypothyroidism should result in the resolution of symptoms such as fatigue, cold intolerance, and other hypothyroid-related manifestations. Rapid weight loss and hyperthyroid symptoms would not be expected outcomes of proper thyroid replacement therapy.

HESI Exit Question: A 45-year-old patient presents with fatigue, weight gain, and cold intolerance. Laboratory tests reveal elevated thyroid-stimulating hormone (TSH) levels and low levels of free thyroxine (FT4). What is the most likely diagnosis for this patient?

  • A) Hyperthyroidism
  • B) Euthyroidism
  • C) Hashimoto’s thyroiditis
  • D) Graves’ disease

Correct Answer: C) Hashimoto’s thyroiditis

Explanation: Hashimoto’s thyroiditis is an autoimmune condition that leads to hypothyroidism. Elevated TSH and low FT4 levels are characteristic of this condition. Patients often present with fatigue, weight gain, and cold intolerance.

HESI Exit Question: A 50-year-old patient with hypothyroidism is prescribed levothyroxine (Synthroid). What important counseling should the nurse provide regarding levothyroxine therapy?

  • A) Take it with calcium supplements to enhance absorption.
  • B) It can be taken with any food or beverage for convenience.
  • C) Take it on an empty stomach in the morning for optimal absorption.
  • D) Discontinue the medication if symptoms improve to avoid overdose.

Correct Answer: C) Take it on an empty stomach in the morning for optimal absorption.

Explanation: Levothyroxine should be taken on an empty stomach in the morning to maximize absorption. Calcium supplements, food, and certain medications can interfere with its absorption.

HESI Exit Question: A 55-year-old patient with hypothyroidism reports constipation and muscle cramps. Which laboratory finding is associated with these symptoms in hypothyroidism?

  • A) Elevated TSH levels
  • B) Elevated FT4 levels
  • C) Normal TSH and FT4 levels
  • D) Low TSH levels

Correct Answer: A) Elevated TSH levels

Explanation: Constipation and muscle cramps are common symptoms of hypothyroidism. Elevated TSH levels indicate insufficient thyroid hormone production, contributing to these symptoms.

HESI Exit Question: A 40-year-old patient with hypothyroidism is started on levothyroxine therapy. After several weeks, the patient’s symptoms persist, and laboratory results show no improvement. What should the nurse consider as a possible cause?

  • A) Patient non-compliance with medication
  • B) The need for a higher levothyroxine dose
  • C) Resolution of hypothyroidism
  • D) Incorrect diagnosis of hyperthyroidism

Correct Answer: A) Patient non-compliance with medication

Explanation: Non-compliance with levothyroxine therapy can lead to persistent symptoms in hypothyroid patients. The nurse should assess the patient’s adherence to the prescribed regimen.

HESI Exit Question: A 60-year-old patient with hypothyroidism is prescribed thyroid desiccated (Armour Thyroid). What unique feature does thyroid desiccated have compared to synthetic thyroid medications?

  • A) It is less expensive.
  • B) It has a more extended duration of action.
  • C) It contains both T3 and T4 hormones.
  • D) It has fewer side effects.

Correct Answer: C) It contains both T3 and T4 hormones.

Explanation: Thyroid desiccated contains both T3 and T4 hormones, providing a combination that may be beneficial for some patients compared to synthetic thyroid medications, which typically contain only T4 (levothyroxine).

HESI Exit Question: A 30-year-old patient presents with weakness, fatigue, and hyperpigmentation of the skin. Laboratory tests reveal low levels of cortisol and elevated adrenocorticotropic hormone (ACTH). What is the most likely diagnosis for this patient?

  • A) Cushing’s syndrome
  • B) Addison’s disease
  • C) Grave’s disease
  • D) Hypothyroidism

Correct Answer: B) Addison’s disease

Explanation: Addison’s disease is characterized by adrenal insufficiency, resulting in low cortisol levels and elevated ACTH levels. The hyperpigmentation of the skin, known as bronzing, is a characteristic feature of this condition.

HESI Exit Question: A 35-year-old patient with Addison’s disease is experiencing an adrenal crisis. What is the immediate nursing intervention for this patient?

  • A) Administering a high dose of corticosteroids
  • B) Administering a high dose of insulin
  • C) Providing fluid and electrolyte replacement
  • D) Administering a beta-blocker

Correct Answer: A) Administering a high dose of corticosteroids

Explanation: An adrenal crisis in Addison’s disease requires immediate administration of a high dose of corticosteroids (e.g., hydrocortisone) to replace deficient hormones and prevent life-threatening complications.

HESI Exit Question: A 40-year-old patient with Addison’s disease is prescribed lifelong corticosteroid replacement therapy. What important patient education should the nurse provide regarding medication management?

  • A) Take corticosteroids with food to reduce gastric irritation.
  • B) Discontinue medication if symptoms improve to avoid overdose.
  • C) Skip doses if experiencing side effects to minimize risks.
  • D) Corticosteroids should be taken at bedtime for best results.

Correct Answer: A) Take corticosteroids with food to reduce gastric irritation.

Explanation: Taking corticosteroids with food can help reduce gastric irritation and minimize the risk of gastrointestinal side effects. Patients should not discontinue medication without medical guidance.

HESI Exit Question: A 45-year-old patient with Addison’s disease is at risk of adrenal crisis during times of illness or stress. What should the nurse instruct the patient to do in such situations?

  • A) Double the daily corticosteroid dose immediately.
  • B) Avoid any medications, including corticosteroids, during illness.
  • C) Seek medical attention promptly and increase corticosteroid dose as prescribed.
  • D) Reduce fluid intake to prevent fluid overload.

Correct Answer: C) Seek medical attention promptly and increase corticosteroid dose as prescribed.

Explanation: In times of illness or stress, patients with Addison’s disease should seek medical attention promptly and follow prescribed instructions for increasing corticosteroid doses to prevent adrenal crisis.

HESI Exit Question: A 50-year-old patient with Addison’s disease is admitted with symptoms of hyperkalemia. What nursing interventions should be prioritized for this patient?

  • A) Administering potassium supplements as prescribed.
  • B) Monitoring for hypotension and providing intravenous fluids.
  • C) Restricting dietary sodium intake.
  • D) Administering diuretics to eliminate excess potassium.

Correct Answer: B) Monitoring for hypotension and providing intravenous fluids.

Explanation: Hyperkalemia can lead to life-threatening arrhythmias in patients with Addison’s disease. Prioritizing monitoring for hypotension and providing intravenous fluids to correct electrolyte imbalances is essential.